Anda di halaman 1dari 80

Respiratory Physiology

Respiratory Muscles
1. A healthy, 45-year-old man is reading the newspaper. Which of the following muscles are used
for quiet breathing?
(A) Diaphragm and external intercostals
(B) Diaphragm and internal intercostals
(C) Diaphragm only
(D) Internal intercostals and abdominal recti
(E) Scaleni
(F) Sternocleidomastoid muscles
2. A healthy, 25-year-old medical student participates in a 10-km charity run for the
American Heart Association. Which of the following muscles does the student use
(contract) during expiration?
(A) Diaphragm and external intercostals
(B) Diaphragm and internal intercostals
(C) Diaphragm only
(D) Internal intercostals and abdominal recti
(E) Scaleni
(F) Sternocleidomastoid muscles

Lung Compartments
Question 3 and 4

Respiratory Physiology
3. A 27-year-old man is breathing quietly. He then inhales as much air as possible and exhales as
much air as he can, producing the spirogram shown in the previous figure. What is his expiratory
reserve volume (in liters)?
(A) 2.0

(B) 2.5

(C) 3.0

(D) 3.5

(E) 4.0

(F) 5.0

4. A 22-year-old woman inhales as much air as possible and exhales as much air as she can
producing the spirogram shown in the figure. A residual volume of 1.0 L was determined using
the helium dilution technique. What is her functional residual capacity (in liters)?
(A) 2.0

(B) 2.5

(C) 3.0

(D) 3.5

(E) 4.0

(F) 5.0

5. A spirometer can be used to directly measure which of the following?


(A) Functional residual capacity
(B) Peak flow rate
(C) Residual volume

(D) Total lung capacity


(E) Vital capacity

6. Which of the following lung volumes or capacities can be measured by spirometry?


(A) Functional residual capacity (FRC)
(B) Physiologic dead space
(C) Residual volume (RV)

(D) Total lung capacity (TLC)


(E) Vital capacity (VC)

7. The various lung volumes and capacities include the total lung volume (TLC), vital capacity
(VC), inspiratory capacity (IC), tidal volume (VT), expiratory capacity (EC), expiratory reserve
volume (ERV), inspiratory reserve volume (IRV), functional residual capacity (FRC), and
residual volume (RV). Which of the following lung volumes and capacities can be measured
using direct spirometry without additional methods?
(A)
(B)
(C)
(D)
(E)

TLC
No
No
No
Yes
Yes

VC
No
Yes
Yes
Yes
Yes

IC
Yes
Yes
Yes
Yes
Yes

VT
No
Yes
Yes
Yes
Yes

EC ERV IRV FRC RV


Yes No Yes No No
Yes Yes Yes No No
Yes Yes Yes Yes No
Yes Yes Yes No Yes
Yes Yes Yes Yes Yes

Respiratory Physiology
8. Which volume remains in the lungs after a tidal volume (TV) is expired?
(A)
(B)
(C)
(D)

Tidal volume (TV)


Vital capacity (VC)
Expiratory reserve volume (ERV)
Residual volume (RV)

(E) Functional residual capacity (FRC)


(F) Inspiratory capacity
(G) Total lung capacity

9. The volume of gas in the lungs at the end of a normal expiration is referred to as the
(A) Residual volume (RV)
(B) Expiratory reserve volume (ERV)
(C) Functional residual capacity (FRC)

(D) Inspiratory reserve volume (IRV)


(E) Total lung capacity (TLC)

10. When all ventilatory muscles are relaxed, the volume in the lungs is at:
(A) Residual volume (RV)
(B) Expiratory reserve volume (ERV)
(C) Functional residual capacity (FRC)

(D) Inspiratory reserve volume (IRV)


(E) Total lung capacity (TLC)

11. A person starts to breathe from a 12 L spirometer containing 10% helium at the end of a passive
expiration. If, after several minutes, the helium concentration in the spirometer falls to 8% this
persons functional residual capacity (FRC) is approximately:
(A) 1.2 L

(B) 2.4 L

(C) 3.0 L

(D) 4.0 L

(E) 4.8 L

12. Which volume remains in the lungs after a maximal expiration?


(A)
(B)
(C)
(D)

Tidal volume (TV)


Vital capacity (VC)
Expiratory reserve volume (ERV)
Residual volume (RV)

(E) Functional residual capacity (FRC)


(F) Inspiratory capacity
(G) Total lung capacity

13. In a maximal expiration, the total volume expired is


(A)
(B)
(C)
(D)

Tidal volume (TV)


Vital capacity (VC)
Expiratory reserve volume (ERV)
Residual volume (RV)

(E) Functional residual capacity (FRC)


(F) Inspiratory capacity
(G) Total lung capacity

Respiratory Physiology
14. The ventilatory capacity of a patient with respiratory disease is evaluated using spirometry.
Which of the following is the correct spirometric term for the largest exhaled volume that this
patient can generate during the course of pulmonary function testing?
(A) Total lung capacity (TLC)
(B) Inspiratory capacity (IC)
(C) Forced vital capacity (FVC)

(D) Inspiratory reserve volume (IRV)


(E) FEV1

15. A 45-year-old man inhaled as much air as possible and then expired with a maximum effort until
no more air could he expired. This produced the maximum expiratory flow-volume curve shown
in the following diagram. What is the forced vital capacity of this man (in liters)?
(A)
(B)
(C)
(D)
(E)
(F)

1.5
2.5
3.5
4.5
5.5
6.5

16. The vital capacity (VC) is the sum of the


(A) Residual volume (RV), tidal volume, and expiratory reserve volume (ERV)
(B) RV, tidal volume, and inspiratory reserve volume (IRV)
(C) RV, ERV, and IRV
(D) ERV, IRV, and tidal volume
(E) Functional residual capacity (FRC) and inspiratory capacity

Respiratory Physiology
17. In the patients spirometry tracing below, the expiratory reserve volume equals which of the
following?

(A) C

(B) D

(C) E

(D) C + D

(E) E D

18. A 35-year-old man has a vital capacity (VC) of 5 L, a tidal volume (TV) of 0.5 L, an inspiratory
capacity of 3.5 L, and a functional residual capacity (FRC) of 2.5 L. what is his expiratory
reserve volume (ERV)?
(A) 4.5 L
(B) 3.9 L

(C) 3.6 L
(D) 3.0 L

(E) 2.5 L
(F) 2.0 L

(G) 1.5 L

19. A patient has a dead space of 150 ml, functional residual capacity of 3 L, tidal volume of 650 ml,
expiratory reserve volume of 1.5 L, total lung capacity of 8 L, and respiratory rate of 15
breaths/min. What is the residual volume?
(A) 500 ml
(B) 1000 ml

(C) 1500 ml
(D) 2500 ml

(E) 6500 ml

20. At which one of the following lung volumes/capacities is transpulmonary pressure at its greatest?
(A) Residual volume
(B) Functional residual capacity
(C) Total lung capacity
(D) When breathing at rest
(E) After taking a deep breath prior to diving into water

Respiratory Physiology
21. All of the following can reduce vital capacity (VC) EXCEPT
(A) Decreased total lung capacity (TLC)
(B) Increased residual volume (RV)
(C) Weakness of the inspiratory muscles

(D) Weakness of the expiratory muscles


(E) Decreased alveolar surface tension

Questions 22-26 are based on the following graph, which shows a normal respiratory cycle
followed by a maximal inspiration, a maximal forced expiration, and another normal respiratory
cycle.

22. Select the lettered arrow that corresponds to vital capacity (VC)

23. Select the lettered arrow that corresponds to forced expiratory volume in 1 second (FEV1)

24. Select the lettered arrow that corresponds to functional residual capacity (FRC)

25. Select the lettered arrow that corresponds to total lung capacity (TLC)

26. Select the lettered arrow that corresponds to residual volume (RV)

Respiratory Physiology
Lung Mechanics
27. Which one of the following statements best characterizes lung compliance?
(A)
(B)
(C)
(D)
(E)

It is equivalent to P/V.
It is inversely related to the elastic recoil properties of the lung.
It decreases with advancing age.
It increases when there is a deficiency of surfactant.
It increases in patients with pulmonary edema

28. Normally, intrapleural pressure is negative throughout a tidal inspiration and expiration because
of which of the following?
(A) The lungs have the tendency to recoil outward throughout a tidal breath.
(B) The chest wall has the tendency to recoil inward throughout a tidal breath.
(C) The intact pleura causes the lungs and chest wall to recoil away from each other throughout
a tidal breath.
(D) The intact pleura causes the lungs and chest wall to recoil in the same direction throughout a
tidal breath.
(E) There is always a small leak in the visceral pleura causing some air to escape into the pleural
space during a tidal breath
29. Which of the following regarding the transmural pressure for the lungs is true?
(A) It is always negative
(B) It is equal to the interpleural pressure minus the atmospheric pressure (PPL PB)
(C) It is equal to the interpleural pressure minus the alveolar pressure (PPL PA)
(D) It is equal to the alveolar pressure minus the interpleural pressure (PA PPL)
(E) It is independent of lung volume when the muscles are relaxed
30. Which of the following is true during inspiration?
(A)
(B)
(C)
(D)
(E)

Intrapleural pressure is positive


The volume in the lungs is less than the functional residual capacity (FRC)
Alveolar pressure equals atmospheric pressure
Alveolar pressure is higher than atmospheric pressure
Intrapleural pressure is more negative than it is during expiration

31. During inspiration, as the diaphragm contracts, the pressure in the interpleural space becomes
(A) Equal to zero
(B) More positive

(C) More negative


(D) Equal to the PA

(E) Equal to the Patm

Respiratory Physiology
32. During inspiration (as compared to expiration):
(A)
(B)
(C)
(D)
(E)

Intrapleural pressure is increasing


Lung recoil is increasing
Abdominal muscles are normally contracting
Both A and B are correct
All of the above are correct

33. At the end of inhalation, with an open glottis, the pleural pressure is
(A) Greater than atmospheric pressure
(B) Equal to atmospheric pressure
(C) Less than alveolar pressure

(D) Equal to alveolar pressure


(E) Greater than alveolar pressure

34. Taking a deep inspiration to total lung capacity causes which of the following cardiopulmonary
function variables to increase?
(A) Alveolar surface tension
(B) Airway resistance
(C) Elastic recoil of the lung

(D) Intrapleural pressure


(E) Lung compliance

35. During a forced expiration, a patient generates an intrapleural pressure of 20 mm Hg. The
patients equal pressure point will move closer to the mouth and forced expiratory volume will
increase if there is an increase in which of the following?
(A) Inspired lung volume
(B) Lung compliance
(C) Airway resistance

(D) Expiratory effort


(E) Airway smooth muscle tone

36. If a person inspires maximally, closes his glottis, and contracts his expiratory muscles as hard as
he can (Valsalva maneuver):
(A)
(B)
(C)
(D)
(E)

Intrapleural pressure is higher than alveolar pressure


Intrapleural pressure and lung recoil both act to increase pressure within the alveoli
Intrapleural pressure is below atmospheric at the apex but above atmospheric at the base
All of the above are correct
None of the above are correct

Respiratory Physiology
37. A 46 year old male during quiet breathing has an intrapleural pressure is about 5 cm H2O at the
start of inspiration (relative to atmospheric pressure). As inspiration proceeds to a full normal
tidal breath, transmural pressure between the alveoli and the pleural space reaches
approximately:
(A) 10 cm H2O
(B) 1 cm H2O

(C) 0 cm H2O
(D) + 1 cm H2O

(E) + 10 cm H2O

38. The pleural pressure of a normal 56-year-old woman is approximately 5 cm H2O during resting
conditions immediately before inspiration (i.e., at functional residual capacity). What is the
pleural pressure (in cm H2O) during inspiration?
(A) + 1

(B) + 4

(C) 0

(D) 3

(E) 7

39. The alveolar pressure of a normal 77-year-old woman is approximately 1 cm H2O during
expiration. What is the alveolar pressure during inspiration (in cm H2O)?
(A) + 0.5

(B) + 1

(C) + 2

(D) 0

(E) 1

(F) 5

40. A 28-year-old male oboe player in the symphony is referred to a pulmonologist due to increasing
exertional dyspnea while playing. A complete pulmonary function study is ordered, and the
patient is instructed to swallow an esophageal balloon for estimating changes in intrapleural
pressures at various lung volumes. The figure below illustrates the change in the patients
intrapleural pressure during a single tidal breath. At which point on the diagram is inspiratory
airflow the greatest?

Respiratory Physiology
41. Maximal inspiratory gas flow occurs when the
(A) Lung volume approaches total lung capacity (TLC)
(B) Lung volume approaches residual volume (RV)
(C) Alveolar pressure is most negative
(D) Interpleural pressure is approximately 5 cm H2O
(E) Abdominal muscles are maximally contracted
Use the following diagram to answer the next 2 questions.

Respiratory Physiology
42. At what time is transpulmonary pressure greatest?
(A) 0 seconds
(B) 1 seconds

(C) 2 seconds
(D) 3 seconds

(E) 4 seconds

43. If the graph holds true for a patient breathing over a period of a minute what is the closest
approximate to their alveolar minute ventilation (assuming a dead space of 200 ml).
(A) 2.5 L/min
(B) 4 L/min
(C) 6.5 L/min

(D) 8 L/min
(E) Cannot be determined from this graph.

44. A man inspires 1000 ml from a spirometer. The intrapleural pressure was 4 cm H2O before
inspiration and 12 cm H2O the end of inspiration. What is the compliance of the lungs?
(A) 50 ml/cm H2O
(B) 100 ml/cm H2O

(C) 125 ml/cm H2O


(D) 150 ml/cm H2O

(E) 250 ml/cm H2O

45. A 22-year old woman has a pulmonary compliance of 0.2 L/cm H2O and a pleural pressure of 4
cm H2O. What is the pleural pressure (in cm H2O) when the woman inhales 1.0 L of air?
(A) 6

(B) 7

(C) 8

(D) 9

(E) 10

46. A liquid-ventilated lung compared to a gas-ventilated lung


(A) Has A reduced airway resistance
(B) Has increased residual volume
(C) Has a more pronounced hysteresis

(D) Is more compliant


(E) Requires greater pressure to inflate

47. Which one of the following statements regarding the compliance of the respiratory system is
true?
(A) It is greater than the compliance of the chest wall
(B) It is greater than the compliance of the lungs
(C) It is equal to the compliance of the chest wall
(D) It is equal to the compliance of the lungs
(E) It is less than the compliance of the chest wall

Respiratory Physiology
48. Which of the following is illustrated in the graph showing volume versus pressure in the
lung-chest wall system?

(A) The slope of each of the curves is resistance


(B) The compliance of the lungs alone is less than the compliance of the lungs plus chest wall
(C) The compliance of the chest wall alone is less than the compliance of the lungs plus chest
wall
(D) When airway pressure is zero (atmospheric), the volume of the combined system is the
functional residual capacity (FRC)
(E) When airway pressure is zero (atmospheric), intrapleural pressure is zero
49. The respiratory system is at the equilibrium position in all of the following conditions
EXCEPT
(A) At the end of a normal expiration
(B) When the transrespiratory pressure is zero
(C) When lung recoil is balanced by chest wall expansion
(D) When lung volume is at residual volume (RV)
(E) When the respiratory muscles are relaxed and the airway is open

Respiratory Physiology
50. The diagram below shows three different compliance curves (S, T, and U) for isolated lungs
subjected to various transpulmonary pressures. Which of the following best describe the relative
compliances for the three curves?
(A) S < T < U
(B) S < T > U
(C) S = T = U
(D) S > T < U
(E) S > T > U

51. The figure below represents the inflation pressure-volume curve of three different lungs. If the
middle curve represents a normal lung, which of the following statements is correct?

(A) The compliance of lungs A and B are both greater than normal.
(B) Lung A is more compliant than normal, and at any lung volume recoil will be greater than in
normal lungs.
(C) Lung B is more compliant than normal, and at any given lung volume recoil will be greater
than in normal lungs.
(D) Lung A is more compliant than normal, and for a given change in surrounding pressure, a
greater change in lung volume will occur than in normal lungs.
(E) Lung B is more compliant than normal, and for a given change in surrounding pressure, a
greater change in lung volume will occur than in normal lungs.

Respiratory Physiology
52. The following pressure-volume curves were obtained from subjects X and Y during quiet
breathing at a rate of 14/min. What conclusion can you draw from these curves? Subject X has
the:
(A) Higher pulmonary compliance
(B) Higher tidal volume
(C) Higher pulmonary compliance and tidal
volume
(D) Lower pulmonary compliance
(E) Lower pulmonary compliance and tidal
volume

53. A patient comes into your office complaining of shortness of breath. He weighs 360 lbs is a
non-smoker and has had no medical history of relevance. Which of the following changes, in
any, would you expect to occur?
(A) An increase in FRC due to an increase in lung compliance
(B) A decrease in FRC due to a decrease in lung compliance
(C) A decrease in FRC due to a decrease in chest wall recoil
(D) An increase in FRC due to an increase in chest wall recoil
(E) None of the above would be observed
54. A 37-year-old woman is admitted to the hospital with severe kyphoscoliosis and respiratory
muscle weakness. Which of the following physiological variables is most likely decreased in this
patient?
(A) Airway resistance
(B) Alveolar surface tension
(C) Arterial carbon dioxide tension

(D) Chest wall compliance


(E) FEV1/FVC

Respiratory Physiology
Dynamic Airway Compression
55. An emphysematic patient comes to your office complaining of difficulty in breathing. You
encourage him to exhale through pursed lips as this will help him by:
(A) Moving the EPP closer to his alveoli
(B) Increasing the compliance of his lungs
(C) Keeping the oxygen tube in his mouth
(D) Decreasing the pressure gradient between the alveoli and trachea
(E) Decreasing the compliance of his lungs
56. The maximum expiratory flow-volume curve shown in the following diagram is used as a
diagnostic tool for identifying obstructive and restrictive lung diseases. At which of the following
points on the curve does airway collapse limit maximum expiratory air flow?

Respiratory Physiology
57. At which point on the flow-volume loop pictured below will airflow remain constant despite an
increased respiratory effort?

58. During the effort-independent portion of a forced vital capacity (EVC) maneuver, the expiratory
flow rate
(A) Varies as a function of the interpleural pressure
(B) Is limited by compression of the airways
(C) Depends on the alveolar pressure
(D) Is maximal for that individual
(E) Is constant
Surfactant
59. If surfactant is absent from the alveoli, the lungs:
(A) Airway resistance will decrease
(B) Compliance will increase
(C) Compliance will decrease

(D) O2 diffusing capacity will increase


(E) Recoil will decrease

Respiratory Physiology
60. A preterm infant has a surfactant deficiency. Without surfactant, many of the alveoli collapse at
the end of each expiration, which in turn leads to pulmonary failure. Which of the following sets
of changes are present in the preterm infant, compared to a normal infant?

(A)
(B)
(C)
(D)
(E)
(F)
(G)

Alveolar Surface Tension Pulmonary Compliance


Decreased
Decreased
Decreased
Increased
Decreased
No change
Increased
Decreased
Increased
Increased
Increased
No change
No change
No change

61. Comparing a premature infant with respiratory distress syndrome to a normal full-term infant,
how do lung compliance and surfactant levels compare?

(A)
(B)
(C)
(D)
(E)
(F)

Compliance in Preterm
Surfactant in Preterm
Compared to Full Term Infant Compared to Full Term Infant

62. A lack of normal surfactant, as occurs in infants with respiratory distress syndrome (RDS),
results in
(A) Increased lung compliance
(B) Stabilization of alveolar volume
(C) Increased retractive force of the lungs
(D) Reduced alveolar-arterial O2 tension difference
(E) Decreased filtration forces in the pulmonary capillaries
63. A deficiency of pulmonary surfactant would:
(A) Decrease surface tension in the alveoli
(B) Decrease the change in intrapleural pressure required to achieve a given tidal volume
(C) Decrease lung compliance
(D) Decrease the work of breathing
(E) Increase functional residual capacity (FRC)

Respiratory Physiology
64. An infant born prematurely in gestational week 25 has neonatal respiratory distress syndrome.
Which of the following would be expected in this infant?
(A)
(B)
(C)
(D)
(E)

Arterial PO2 of 100 mm Hg


Collapse of the small alveoli
Increased lung compliance
Normal breathing rate
Lecithin:sphingomyelin ratio of greater than 2:1 in amniotic fluid

65. A 35-year-old woman with gestational diabetes develops hypertension and preeclampsia,
requiring the preterm delivery of her fetus of 30-weeks gestation. The mother is given two doses
of betamethasone, 12 mg, intramuscularly, 24 hours apart. Which of the following is the purpose
of the antenatal steroid therapy?
(A)
(B)
(C)
(D)
(E)

Increase fetal PO2


Increase blood flow to the fetal lungs
Shift the fetal oxyhemoglobin dissociation curve to the right
Increase blood flow from the right atrium into the left atrium across the foramen ovale
Increase the lecithin/sphingomyelin ratio in the amniotic fluid

66. A 27-year-old man develops adult respiratory distress syndrome (ARDS) after near-drowning.
Conventional mechanical ventilation on 100% O2 together with inhaled nitric oxide do not
provide sufficient oxygenation. Porcine surfactant is instilled via fiberoptic bronchoscope, and
the PaCO2, fraction of inspired oxygen (FIO2), and shunting improve impressively. The
improvements in respiratory function occurred because surfactant increased which of the
following?
(A)
(B)
(C)
(D)
(E)

Bronchiolar smooth muscle tone


The pressure gradient needed to inflate the alveoli
Lung compliance
Alveolar surface tension
The work of breathing

Respiratory Physiology
67. A hospitalized patient has tachypnea and significantly labored respirations requiring mechanical
ventilation. Based on the pressure-volume curve of the lungs shown as curve Z in the figure
below, which of the following is the most likely diagnosis for the patient?
(A)
(B)
(C)
(D)

Asthma
Emphysema
Dyspnea with aging
Newborn with lecithin to sphingomyelin
(L/S) ratio greater than 2
(E) Pulmonary edema

Airway Resistance
68. Which of the following is the site of highest airway resistance?
(A) Trachea
(B) Largest bronchi

(C) Medium-sized bronchi


(D) Smallest bronchi

(E) Alveoli

69. The major area of airway resistance during breathing is located in the
(A) Oropharynx
(B) Trachea and large bronchi
(C) Intermediate-sized bronchi

(D) Bronchioles < 2 mm in diameter


(E) Alveoli

70. The resistance of the pulmonary tree is so low that a 1 cm of water pressure gradient is sufficient
to cause normal air flow during resting conditions. Which of the following often has a substantial
resistance during pulmonary disease states that can limit alveolar ventilation?
(A) Alveoli
(B) Bronchioles

(C) Large bronchi


(D) Small bronchi

(E) Trachea

Respiratory Physiology
71. Airway resistance is greater
(A)
(B)
(C)
(D)
(E)

With laminar flow than with turbulent flow


At lower values for Reynolds number
During inspiration compared to expiration
At low lung volumes compared to high lung volumes
In the total cross section of the small airways compared to the total cross section of the
central airways

72. Resistance in the airways of the lungs decreases:


(A)
(B)
(C)
(D)
(E)

In response to sympathetic nerve stimulation


In response to parasympathetic nerve stimulation
In response to a decrease in alveolar PCO2
As the diameter of the air tubes decreases
As the velocity of airflow increases

73. Airway resistance can be reduced by


(A) Increasing vagal impulses to the lungs
(B) Administering a p-adrenergic blocking drug
(C) Decreasing the radial traction exerted by lung tissue
(D) Performing a maximal forced expiration
(E) Increasing lung volume
74. The following diagram shows pulmonary airway resistance expressed as a function of pulmonary
volume. Which relationship best describes the normal lung?

Respiratory Physiology
75. Which of the following will increase as a result of stimulating cholinergic receptors on the
bronchial smooth muscle?
(A) Lung compliance
(B) Airway diameter
(C) Elastic work of breathing

(D) Resistive work of breathing


(E) Anatomic dead space

76. The respiratory passageways have smooth muscle in their walls. Which of the following best
describes the effect of acetylcholine and epinephrine on the respiratory passageways?
(A)
(B)
(C)
(D)
(E)
(F)
(G)
(H)

Acetylcholine Epinephrine
Constrict
Constrict
Constrict
Dilate
Constrict
No effect
Dilate
Constrict
Dilate
Dilate
Dilate
No effect
No effect
Constrict
No effect
Dilate

77. A 58-year-old woman experiences an acute exacerbation of asthma, which causes her breathing
to become labored and faster. As a result, which of the following changes in airflow is expected?
(A)
(B)
(C)
(D)
(E)

Flow in the trachea and upper airways will become more laminar.
The pressure gradient required for airflow will increase.
The resistance to airflow will decrease.
The resistance to airflow will increase linearly with the decrease in airway radius
Reynolds number will decrease.

78. Which of the following will decrease the oxygen consumption of the respiratory muscles?
(A) A decrease in lung compliance
(B) A decrease in airway resistance
(C) A decrease in the diffusing capacity of the lung
(D) An increase in the rate of respiration
(E) An increase in tidal volume

Respiratory Physiology
Ventilation
79. Which of the following formulas is correct?
(A)
(B)
(C)
(D)

Vital capacity = inspiratory volume + expiratory reserve volume


Dead air space = resting tidal volume + residual volume
Alveolar ventilation = (respiratory rate) (tidal volume dead air space)
Vital capacity = inspiratory reserve volume + resting tidal volume + expiratory reserve
volume + residual volume
(E) Inspiratory reserve volume = vital capacity resting tidal volume
80. Alveolar ventilation is equal to the
(A) Dead space ventilation
(B) Tidal volume times respiratory rate
(C) Minute ventilation
(D) Minute ventilation minus dead space ventilation
(E) CO2 production/min
81. A healthy 65-year-old man with a tidal volume (TV) of 0.45 L has a breathing frequency of l6
breaths/min. His arterial PCO2 is 41 mm Hg, and the PCO2 of his expired air is 35 mm Hg. What is
his alveolar ventilation?
(A) 0.066 L/min
(B) 0.38 L/min

(C) 5.0 L/min


(D) 6.14 L/min

(E) 8.25 L/min

82. A fat patient scheduled for abdominal surgery is sent for preoperative evaluation and testing. His
chest x-ray is normal, and pulmonary function results on room air show:
Tidal volume = 600 mL
Respiratory rate = 12/min
Vital capacity = 5,000 mL

PaO2 = 90 mm Hg
PaCO2 = 40 mm Hg
PECO2 = 28 mm Hg

The volume of the patients physiological dead space, determined by applying the Bohr equation,
equals which of the following?
(A) 7,200 mL
(B) 420 mL

(C) 180 mL
(D) 150 mL

(E) 0.3 mL

Respiratory Physiology
83. A 125-lb, 40-year-old woman with a history of nasal polyps and aspirin sensitivity since
childhood presents to the emergency department with status asthmaticus and hypercapnic
respiratory failure. She requires immediate intubation and is placed on a mechanical ventilator on
an FIO2 of 40%, a control rate of 15 breaths/min, and a tidal volume of 500 mL. Which of the
following is her approximate alveolar ventilation?
(A) 375 mL/min
(B) 3,500 mL/min

(C) 5,250 mL/min


(D) 5,625 mL/min

(E) 7,500 mL/min

84. A man breathing at a frequency of 20 breaths per minute has a minute ventilation of 8 L/min and
a dead space of 150 mL. His alveolar ventilation is:
(A) 250 mL/min
(B) 400 mL/min

(C) 2,500 mL/min


(D) 3,000 mL/min

(E) 5,000 mL/min

85. A patient is on a ventilator adjusted for an inspiratory tidal volume of 1 L at a frequency of 10


b/min. If the patients anatomic dead space is 200 mL and the machines dead space 50 mL, the
patient's alveolar ventilation is:
(A) 10 L/min
(B) 8.0 L/min

(C) 7.5 L/min


(D) 5 L/min

(E) Not determinable from


the information given

86. A patient has a dead space of 150 ml. functional residual capacity of 3 L, tidal volume of 650
ml, expiratory reserve volume of 1.5 L, a total lung capacity of 8 L, respiratory rate of 15
breaths/min. What is the alveolar ventilation?
(A) 5 L/min

(B) 7.5 L/min

(C) 6.0 L/min

(D) 9.0 L/min

87. A medical student waiting to do her first patient interview at the clinical skills center becomes
very anxious and increases her rate of alveolar ventilation. If her rate of CO2 production remains
constant, which of the following will decrease?
(A) pH
(B) PaO2

(C) PaCO2
(D) V/Q

(E) Alveolar-arterial
difference

88. Increasing the tidal volume while keeping everything else constant increases the
(A) Dead space ventilation
(B) Functional residual capacity (FRC)
(C) Inspiratory capacity

(D) Alveolar ventilation


(E) Alveolar CO2 tension

PO2

Respiratory Physiology
89. Which of the following could produce a decrease in alveolar ventilation with no change in total
ventilation?
(A)
(B)
(C)
(D)
(E)

A decreased functional residual capacity


A decreased respiratory rate and tidal volume
An increased respiratory rate and decreased tidal volume
A decreased respiratory rate and increased tidal volume
An increased respiratory rate and tidal volume

90. An experiment is conducted in two individuals (subjects T and V) with identical tidal volumes
(1000 ml), dead space volumes (200 ml), and ventilation frequencies (20 breaths per minute).
Subject T doubles his tidal volume and reduces his ventilation frequency by 50%. Subject V
doubles his ventilation frequency and reduces his tidal volume by 50%. Which of the following
best described the total ventilation (also called minute ventilation) and alveolar ventilation of
subjects T and V?
(A)
(B)
(C)
(D)
(E)
(F)
(G)

Total ventilation Alveolar ventilation


T<V
T=V
T<V
T>V
T=V
T<V
T=V
T=V
T=V
T>V
T>V
T<V
T>V
T=V

91. The concentration of CO2 is lowest in the:


(A) Anatomical dead space at end inspiration
(B) Anatomical dead space at end expiration
(C) Alveoli at end inspiration

(D) Alveoli at end expiration


(E) Blood in the pulmonary veins

92. The major sign of hypoventilation is


(A) Cyanosis
(B) Increased airway resistance
(C) Hypercapnia

(D) Dyspnea
(E) Hypoxia

Respiratory Physiology
93. A 62-year-old man is known to have chronic lung disease and hypercapnia. He needs a major
operation to remove an intestinal tumor. To ensure that he has adequate alveolar ventilation while
being anesthetized, which of the following should be available?
(A) Tank of 100% O2
(B) Tank of 95% O2, 5% CO2
(C) Mechanical respirator

(D) Cardiac defibrillator


(E) Electrocardiograph

94. A reduction in local alveolar ventilation is associated with


(A) An increase in regional pulmonary blood flow
(B) A decrease in regional alveolar CO2 tension
(C) A decrease in regional alveolar O2 tension
(D) An increase in regional tissue pH
(E) An increase in capillary hemoglobin saturation

Alveolar-Blood Gas Exchange


95. The volume of N2 dissolved in body fluids is greatest while breathing which of the following gas
mixtures?
(A) Air at sea level
(B) Air at an altitude of 15,000 feet
(C) 20% O2, 20% N2, 60% He, while scuba diving at 2 atm of pressure
(D) 20% O2, 30% N2, 50% He, while scuba diving at 2 atm of pressure
(E) 20% O2, 10% N2, 70% He, while scuba diving at 5 atm of pressure
96. A person with normal lungs al sea level (760 mm Hg) is breathing 50% oxygen. What is the
approximate alveolar Po2?
(A) 100 mmHg
(B) 159 mmHg

(C) 268 mmHg


(D) 330 mmHg

(E) 380 mmHg

97. Which of the following statements regarding the fraction of O2 in inspired (tracheal) gas is true?
(A) It equals 0.25 at sea level
(B) It decreases as a function of altitude
(C) It varies as a function of the weather
(D) It is less than the fraction of O2 in the atmosphere
(E) It equals the fraction of O2 in the alveoli

Respiratory Physiology
98. The water vapor pressure of alveolar gas at a barometric pressure of 380 mm Hg is: (Assume all
other parameters are same as at sea level.)
(A) 23.5 mm Hg
(B) 47.0 mm Hg

(C) 76.0 mm Hg
(D) 94.0 mm Hg

(E) 105.0 mm Hg

99. Which one of the following statements regarding the CO2 tension in mixed expired gas is true?
(A) It is greater than the alveolar CO2 tension
(B) It is less than the alveolar CO2 tension
(C) It is equal to the alveolar CO2 tension
(D) It is equal to the atmospheric CO2 tension
(E) It is greater than the CO2 tension in venous blood
100. The alveolar PO2 of an individual breathing 30% O2 at an atmospheric pressure of 747 mm Hg
will be (in mm Hg): (assume R = 1.0 and PACO2 = 40 mm Hg)
(A) 224

(B) 210

(C) 170

(D) 164

(E) 100

101. A 56-year-old man presents to the emergency department with severe abdominal pain and a
temperature of 103F The patient is in severe respiratory distress. Moderate amounts of
pulmonary edema fluid are aspirated during suctioning. The patient is placed on a ventilator with
an FIO2 of 0.5 and an arterial blood gas sample reveals a PO2 of 160 mm Hg and a PCO2 of 40 mm
Hg. His alveolar oxygen tension, at a barometric pressure of 747 mm Hg and a respiratory
exchange ratio (R) of 0.8, is approximately what?
(A) 100 mm Hg
(B) 200 mm Hg

(C) 300 mm Hg
(D) 400 mm Hg

(E) 500 mm Hg

102. A 36-year-old man with a history of AIDS and Pneumocystis infection presents to the emergency
department with severe respiratory distress. The patient is placed on a ventilator at a rate of 16,
tidal volume of 600 mL, and FIO2 of 1.0. An arterial blood sample taken 20 minutes later reveals
a PO2 of 350 mm Hg, a PCO2 of 36 mm Hg, and a pH of 7.32. At a barometric pressure of 757 mm
Hg, and assuming a normal respiratory exchange ratio (R) of 0.8, the patients alveolar oxygen
tension is approximately which of the following?
(A) 105 mm Hg
(B) 355 mm Hg

(C) 576 mm Hg
(D) 665 mm Hg

(E) 712 mm Hg

Respiratory Physiology
103. If alveolar PCO2 was originally 40 mm Hg but body temperature increased and CO2 production
doubled while no change occurred in alveolar ventilation, what decrease should occur in alveolar
PO2? (Assume R = 1.0)
(A) 10 mm Hg
(B) 20 mm Hg

(C) 30 mm Hg
(D) 40 mm Hg

(E) No change

104. In which of the following conditions is alveolar Po2 increased and alveolar Pco2 decreased?
(A) Increased alveolar ventilation and unchanged metabolism
(B) Decreased alveolar ventilation and unchanged metabolism
(C) Increased metabolism and unchanged alveolar ventilation
(D) Proportional increase in metabolism and alveolar ventilation
105. An individuals inspired PO2 was 150 mm Hg and his alveolar PCO2 was 40 mm Hg. If this
persons alveolar ventilation then doubled, his alveolar PO2 would be expected to change by:
(assume a new steady state and an R value of 1.0)
(A) 20 mm Hg
(B) 25 mm Hg

(C) 40 mm Hg
(D) 50 mm Hg

(E) No change

106. A persons normal tidal volume is 400 ml with a dead space of 100 ml. The respiratory rate is 12
breaths/min. The person is placed on ventilator for surgery and the tidal volume is 700 ml with a
rate of 12 breaths/min. What is the approximate alveolar Pco2 for this person?
(A) 10 mmHg
(B) 20 mmHg

(C) 30 mmHg
(D) 40 mmHg

(E) 45 mmHg

107. Arterial PO2 is measured under 5 different conditions. The one you would expect to produce the
lowest PaO2 value is the when the individual:
(A) Has just completed 15 minutes of strenuous exercise
(B) Has spent 10 minutes in a pressure chamber which stimulates an altitude of 16,000 feet (4900
meters)
(C) Has a 15% reduction in pulmonary blood flow due to a decreased heart rate
(D) Has been in an environmental chamber for 5 minutes in which the temperature is 100F
(38C) and the relative humidity 95%
(E) Has a minute ventilation of 9.0 liters per minute and a cardiac output of 5.0 liters per minute

Respiratory Physiology
108. A 28-year-old man is admitted to the emergency department multiple fractures suffered in a car
accident. Arterial blood gases are ordered while the patient is breathing room air. After obtaining
the arterial blood sample, the blood-gas technician draws room air into the syringe before
measuring the blood-gas values. How does exposure to room air affect the measured values of
PaO2 and PaCO2?
(A) The measured values of both PaO2 and PaCO2 will be higher than the patients actual values.
(B) The measured values of both PaO2 and PaCO2 will be lower than the patients actual values.
(C) The measured PaO2 will be higher and the measured PaCO2 will be lower than the patients
actual blood gas values.
(D) The measured PaO2 will be lower and the measured PaCO2 will be higher than the patients
actual blood gas values.
(E) The measured values of PaO2 and PaCO2 will accurately reflect the actual values.

Alveolar-Blood Gas Transfer/Diffusion


109. The diffusing capacity of a gas is the volume of gas that will diffuse through a membrane each
minute for a pressure difference of 1 mm Hg. Which of the following gases is often used to
estimate the oxygen diffusing capacity of the lungs?
(A) Carbon dioxide
(B) Carbon monoxide

(C) Cyanide gas


(D) Nitrogen

(E) Oxygen

110. A series of gas mixtures is inhaled by a healthy subject. Which one of the following gases would
diffuse most slowly from the lungs into the blood?
(A) CO2 at PCO2 of 60 mm Hg
(B) CO at a PCO of 0.5 mm Hg

(C) O2 at a PO2 of 130 mm Hg


(D) O2 at a PO2 of 150 mm Hg

111. The diffusion coefficient of O2, as compared with that of CO2, is


(A) Greater because O2 combines with hemoglobin
(B) Less because O2 is less soluble
(C) Greater because of a higher pressure gradient
(D) Less because of the lower molecular weight of O2
(E) Essentially the same

Respiratory Physiology
112. The forces governing the diffusion of a gas through a biological membrane include the pressure
difference across the membrane (P), the cross-sectional area of the membrane (A), the
solubility of the gas (S), the distance of diffusion (d), and the molecular weight of the gas (MW).
Which of the following changes increases the diffusion of a gas through a biological membrane?
(A)
(B)
(C)
(D)
(E)

P
A
S
d
MW
Increase Increase Increase Increase Increase
Increase Increase Increase Increase Decrease
Increase Decrease Increase Decrease Decrease
Increase Increase Increase Decrease Increase
Increase Increase Increase Decrease Decrease

113. Which of the following would be expected to decrease the rate of O2 diffusion across the
alveolar-capillary membrane of the lung?
(A)
(B)
(C)
(D)

A decrease in PO2 difference between the alveolus and pulmonary capillary blood
An increase in the surface area of the alveolar-capillary membrane
An increase in the thickness of the alveolar-capillary membrane
A decrease in O2 solubility in the alveolar membrane

114. A subject inspires a mixture of gases containing CO and holds his breath for 10 seconds. It is
calculated that during the 10 seconds when the subject held his breath, the alveolar PCO is 0.5
mm Hg, and the CO uptake is 25 ml/min. What is the diffusing capacity for CO (D L)? (DL is
uptake of CO in ml/min/mm Hg.)
(A) 12.5

(B) 50

(C) 125

(D) 250

(E) 500

Alveolar-Arterial O2 Gradient
115. A 26-year-old medical student on a normal diet has a respiratory exchange ratio of 0.8. How
much oxygen and carbon dioxide are transported between the lungs and tissues of this student (in
ml gas/100 ml blood)?
(A)
(B)
(C)
(D)
(E)
(F)

Oxygen Carbon Dioxide


4
4
5
3
5
4
5
5
6
3
6
4

Respiratory Physiology
116. A 68-year-old man who has chronic obstructive pulmonary disease (COPD) presents to his
pulmonologist with fatigue, dyspnea at rest, and peripheral edema. His blood gases on room air
are PaO2= 60 mm Hg, PaCO2 = 60 mm Hg, and pH = 7.36. His alveolar-arterial (A-a) O2 gradient,
at a barometric pressure of 760 mm Hg and a respiratory exchange ratio (R) of 0.8, is
approximately what?
(A) 5 mmHg
(B) 10 mm Hg

(C) 15 mmHg
(D) 20 mm Hg

(E) 25 mmHg

117. A patient presents to you in the emergency department with a history of progressive respiratory
distress. You obtain an arterial blood gas (ABG) analysis while the patient is breathing room air
that shows the following: PaO2 = 65, SaO2 = 0.90, PaCO2 = 50, pH = 7.30. The patient's
temperature is 37 degrees C, barometric pressure is 760 mm Hg, Water vapor pressure at body
temperature is 47 mm Hg, the respiratory quotient is 0.8. What is the patient's alveolar-arterial
oxygen tension gradient; P(A-a)O2?
(A) 15 mm Hg
(B) 22 mm Hg

(C) 85 mm Hg
(D) 95 mm Hg

(E) 35 mm Hg

118. Which person would be expected to have the largest A-a gradient?
(A)
(B)
(C)
(D)
(E)

Person with pulmonary fibrosis


Person who is hypoventilating due to morphine overdose
Person at 12,000 feet above sea level
Person with normal lungs breathing 50% O2
Person with normal lungs breathing 100% O2

119. Which of the following causes of hypoxia is characterized by a decreased arterial P O2 and an
increased A-a gradient?
(A) Hypoventilation
(B) Right-to-left cardiac shunt
(C) Anemia

(D) Carbon monoxide poisoning


(E) Ascent to high altitude

Respiratory Physiology
Oxygen, Carbon Dioxide Transport (Hemoglobins)
120. The following diagram shows a normal oxygen- hemoglobin dissociation curve. Which of the
following are approximate values of hemoglobin saturation (% Hb-O2), oxygen partial pressure
(Po2), and oxygen content (O2 content) for oxygenated blood leaving the lungs and reduced
blood returning to the lungs from the tissues?

(A)
(B)
(C)
(D)
(E)

Oxygenated blood
Reduced blood
% Hb-O2 Po2 O2 content % Hb-O2 Po2 O2 content
100
104
15
80
42
16
100
104
20
30
20
6
100
104
20
75
40
15
90
100
16
60
30
12
98
140
20
75
40
15

Respiratory Physiology
121. The diagrams show changes in the partial pressures of oxygen and carbon dioxide as blood flows
from the arterial end to the venous of the pulmonary capillaries. Which diagram best depicts the
normal relationship between Po2 (red line) and Pco2 (green line) during resting conditions?

122. Which of the following oxygen-hemoglobin dissociation curves corresponds to blood during
resting conditions (red line) and blood during exercise (green line)?

Respiratory Physiology
123. A 32-year-old medical student has a fourfold increase in cardiac output during strenuous exercise.
Which of the curves on the following diagram most likely represents the changes in oxygen
tension that occur as blood flows from the arterial end to the venous end of the pulmonary
capillaries in this student?

124. Which of the following oxygen-hemoglobin dissociation curves corresponds to blood


from an adult (red line) and blood from a fetus (green line)?

Respiratory Physiology
Use below diagram for next two questions
A

125. In the hemoglobin-O2 dissociation curves shown above, the shift from curve A to curve B could
be caused by
(A)
(B)
(C)
(D)
(E)

Increased pH
Decreased 2,3-diphosphoglycerate (DPG) concentration
Strenuous exercise
Fetal hemoglobin (HbF)
Carbon monoxide (CO) poisoning

126. The shift from curve A to curve B is associated with


(A)
(B)
(C)
(D)
(E)

Increased P50
Increased affinity of hemoglobin for O2
Impaired ability to unload O2 in the tissues
Increased O2-carrying capacity of hemoglobin
Decreased O2-carrying capacity of hemoglobin

Respiratory Physiology
127. Which of the points on the following figures represent arterial blood in a severely anemic
person?

(A)
(B)
(C)
(D)

Figure 1
D
E
D
E

Figure 2
D
E
E
D

128. A 34-year-old woman is anemic with a blood hemoglobin concentration of 7.1 g/dL. Which of
the following sets of changes has occurred in this woman, compared to normal?
(A)
(B)
(C)
(D)
(E)
(F)
(G)
(H)
(I)

Arterial Po2 Mixed venous Po2 2,3 Diphosphoglycerate


Decreased
Decreased
Increased
Decreased
Decreased
Normal
Decreased
Normal
Decreased
Increased
Decreased
Normal
Increased
Increased
Increased
Increased
Normal
Decreased
Normal
Decreased
Decreased
Normal
Decreased
Increased
Normal
Normal
Normal

Respiratory Physiology
129. Which of the following oxygen-hemoglobin dissociation curves corresponds to normal blood
(red line) and blood containing carbon monoxide (green line)?

130. In the below figure, if curve N represents the oxygen-Hb dissociation curve of a normal
individual, then curve A represents:
N

O2 volume %

Curves N and B have the same P50


Curve A has a lower P50

PO2

(A)
(B)
(C)
(D)
(E)

Polycythemia and B represents anemia


Anemia and B represents polycythemia
Anemia and B represents carbon monoxide poisoning
Carbon monoxide poisoning and B represents anemia
Carbon monoxide poisoning and B represents polycythemia

Respiratory Physiology
131. Which of the following best represents the systemic arterial blood of an individual with anemia?
(A)
(B)
(C)
(D)
(E)

Low PO2, low hemoglobin, normal O2 content


Low PO2, low hemoglobin, low O2 content
Normal PO2, low hemoglobin, low O2 content
Normal PO2, normal hemoglobin, low O2 content
Low PO2, normal hemoglobin, low O2 content

132. Which of the following best characterizes the systemic arterial blood of an individual suffering
from carbon monoxide poisoning?
(A)
(B)
(C)
(D)
(E)

Low [Hb], low O2 content, low PO2


Low [Hb], normal O2 content, low PO2
Low [Hb], low O2 content, normal PO2
Normal [Hb], low O2 content, low PO2
Normal [Hb], low O2 content normal PO2

133. A decrease in blood pH will displace the hemoglobin dissociation curve to the:
(A)
(B)
(C)
(D)

Right and reduce O2 carrying capacity


Right with no change in O2 carrying capacity
Left and reduce O2 carrying capacity
Left with no change in O2 carrying capacity

134. Which of the following will decrease the O2 carrying capacity of blood?
(A) Increased PCO2
(B) Increased temperature

(C) Decrease in pH
(D) Decreased hemoglobin

135. Normally as the alveolar PO2 is increased from 110 to 950 mm Hg, the amount of oxygen:
(A)
(B)
(C)
(D)

Dissolved in plasma increases, amount associated with hemoglobin remains almost constant
Dissolved in plasma increases, amount associated with hemoglobin decreases.
Dissolved in plasma remains constant, amount associated with hemoglobin increases.
Dissolved in plasma remains constant, amount associated with hemoglobin remains constant

136. Most of the carbon dioxide carried by the blood is:


(A) Dissolved as CO2
(B) Plasma bicarbonate
(C) Red blood cell bicarbonate

(D) Carbamino compounds,


(E) Combined with plasma proteins

Respiratory Physiology
137. In a healthy human subject, either at rest or during exercise, most of the carbon dioxide
transported from the peripheral tissues to the lungs is in which of the following forms?
(A) Carbaminohemoglobin
(B) Dissolved CO2

(C) Carbonic acid


(D) Bicarbonate

(E) Carboxyhemoglobin

138. A 17-year-old female was bicycling without a helmet when she fell and hit her head. In the
emergency room, she was not conscious and was receiving ventilator assistance. Her blood gases
follow:
PaO2 = 52 mm Hg
PaCO2 = 75 mm Hg

pH = 7.15
HCO3- = 31 mM

The majority of the CO2 was being transported as


(A) CO2 bound to plasma proteins
(B) CO2 bound to hemoglobin

(C) Bicarbonate ions


(D) Dissolved

139. Carbon dioxide is transported in the blood in the dissolved state, in the form of bicarbonate ion,
and in combination with hemoglobin (carbaminohemoglobin). Which of the following best
describes the quantitative relationship of these three mechanisms for transporting carbon dioxide
in the venous blood under normal conditions (in percentages)?
(A)
(B)
(C)
(D)
(E)
(F)

Dissolved State Bicarbonate Ion Carbaminohemoglobin


7
70
23
70
23
7
23
70
7
7
23
70
70
7
23
23
7
70

Respiratory Physiology
140. Carbon dioxide is transported from the tissues to the lungs predominantly in the form of
bicarbonate ion. Compared to arterial red blood cells, which of the following best describes
venous red blood cells?

(A)
(B)
(C)
(D)
(E)
(F)
(G)
(H)
(I)

Intracellular Chloride Cell Volume


Concentration
Decreased
Decreased
Decreased
Increased
Decreased
No change
Increased
Decreased
Increased
No change
Increased
Increased
No change
Decreased
No change
Increased
No change
No change

141. In the transport of CO2 from the tissues to the lungs, which of the following occurs in venous
blood?
(A)
(B)
(C)
(D)
(E)

Conversion of CO2 and H2O to H+ and HCO3 in the red blood cells (RBCs)
Buffering of H+ by oxyhemoglobin
Shifting of HCO3 into the RBCs from plasma in exchange for Cl
Binding of HCO3 to hemoglobin
Alkalinization of the RBCs

142. The pH of venous blood is only slightly more acidic than the pH of arterial blood because
(A)
(B)
(C)
(D)
(E)

CO2 is a weak base


There is no carbonic anhydrase in venous blood
The H+ generated from CO2 and H2O is buffered by HCO3 in venous blood
The H+ generated from CO2 and H2O is buffered by deoxyhemoglobin in venous blood
Oxyhemoglobin is a better buffer for H+ than is deoxyhemoglobin

Respiratory Physiology
Use below diagram for next 2 questions

143. Which letter on the above graph most closely represents systemic arterial blood?

144. Which letter on the above graph most closely represents mixed venous blood?

145. A person with anemia has a hemoglobin (Hb) concentration of 12 g/dL. He starts exercising and
uses 12 ml O2/dL. What is the mixed venous Po2?
(A) 0 mm Hg
(B) 10 mm Hg

(C) 20 mm Hg
(D) 40 mm Hg

(E) 100 mm Hg

Control of Breathing
146. In a normal individual, respiration (alveolar ventilation) is regulated mainly via:
(A)
(B)
(C)
(D)
(E)

Peripheral chemoreceptors responding to changes in PO2


Peripheral chemoreceptors responding to changes in PCO2
Central chemoreceptors responding to changes in PO2
Central chemoreceptors responding to changes in PCO2
Central chemoreceptors responding to changes in both PCO2 and PO2

Respiratory Physiology
147. Hypercapnia affects respiration primarily by stimulating the
(A) Carotid and aortic bodies
(B) Receptors
(C) Central (medullary) chemoreceptors

(D) Arterial baroreceptors


(E) Hypoglossal nerve

148. The activity of the central chemoreceptors is stimulated by which of the following?
(A)
(B)
(C)
(D)
(E)

An increase in the PCO2 of blood flowing through the brain


A decrease in the PO2 of blood flowing through the brain
An increase in the pH of the CSF
A decrease in the metabolic rate of the surrounding brain tissue
Hypoxemia, hypercapnia, and metabolic acidosis

149. In a healthy, normal human subject, which one of the following physiological mechanisms is
primarily responsible for the regulation of arterial PCO2, whether sleeping, studying, or engaging
in exhaustive physical activity?
(A) Renal regulation of bicarbonate secretion
(B) The erythrocyte chloride shift
(C) Regulation of alveolar ventilation
(D) Shifts in the position of the carbon dioxide dissociation curve
(E) Up-regulation of red blood cell carbonic anhydrase
150. At a fraternity party a 17-year-old male places a paper bag over his mouth and breathes in and
out of the bag. As he continues to breathe into this bag, his rate of breathing continues to increase.
Which of the following is responsible for the increased ventilation?
(A) Increased alveolar Po2
(B) Increased alveolar Pco2

(C) Decreased arterial Pco2


(D) Increased pH

Respiratory Physiology
151. Which diagram best describes the relationship between alveolar ventilation (VA) and arterial
carbon dioxide tension (Pco2) when the Pco2 is changed acutely over a range of 35 to 75 mm Hg?

152. The peripheral chemoreceptors for oxygen (in carotid and aortic bodies) increase their rate of
discharge primarily in response to:
(A)
(B)
(C)
(D)
(E)

A decrease in blood oxygen content


A decrease in the partial pressure of blood oxygen
A decrease in blood H+ concentration
A decrease in dissolved oxygen in cerebral spinal fluid
An increase in percent saturation of hemoglobin with oxygen

153. Hypoxemia produces hyperventilation by a direct effect on the


(A) Phrenic nerve
(B) J receptors
(C) Lung stretch receptors

(D) Medullary chemoreceptors


(E) Carotid and aortic body chemoreceptors

154. Immediately upon arriving at a ski resort located 11,500 feet above sea level, a healthy sea-level
native would experience an increase in resting minute ventilation, most likely due to which of the
following?
(A) The effect of low arterial PO2 on central chemoreceptors
(B) The effect of increased arterial PCO2 on central chemoreceptors
(C) The CNS ischemic reflex
(D) The effect of increased arterial PCO2 on peripheral chemoreceptors
(E) The effect of low arterial PO2 on peripheral chemoreceptors

Respiratory Physiology
155. Which diagram best describes the relationship between alveolar ventilation (VA) and arterial
oxygen tension (Po2) when the Po2 is changed acutely over a range of 0 to 160 mm Hg and the
arterial Pco2 and hydrogen ion concentration remain normal?

156. Both the peripheral and the central chemoreceptors mediate increased ventilation in response to
which of the following?
(A) Acute hypercapnia
(B) Acute metabolic acidosis
(C) Chronic hypercapnia

(D) Chronic hypertension


(E) Chronic hypoxemia

157. An anesthetized male is breathing with no assistance. He is then artificially ventilated for 10 min
at his normal tidal volume but at twice his normal frequency. He is ventilated with a gas mixture
of 60% O2 and 40% N2. The artificial ventilation is stopped and he fails to breathe for several
minutes. This apneic episode is due to which of the following?
(A)
(B)
(C)
(D)
(E)

High arterial Po2 suppressing the activity of the peripheral chemoreceptors


Decrease in arterial pH suppressing the activity of the peripheral chemoreceptors
Low arterial Pco2 suppressing the activity of the medullary chemoreceptors
High arterial Pco2 suppressing the activity of the medullary chemoreceptors
Low arterial Pco2 suppressing the activity of the peripheral chemoreceptors

Respiratory Physiology
158. Reduction of functional hemoglobin associated with anemia, methemoglobinemia, or carbon
monoxide poisoning does not produce an increased respiratory rate because the:
(A) Blood flow to the carotid body is unchanged
(B) Arterial oxygen content is maintained within the normal range
(C) Carotid body chemoreceptors are stimulated
(D) SaO2 of arterial blood is normal
(E) PaO2 of arterial blood is normal
159. Which of the following occurs with carbon monoxide inhalation?
(A)
(B)
(C)
(D)
(E)
(F)

Alveolar Po2 Alveolar Pco2 Peripheral Chemoreceptor Activity

160. A patient with uncontrolled diabetes mellitus experiences an episode of diabetic ketoacidosis.
Which of the following is the most likely mechanism underlying the acute ventilatory response
to such a sudden increase in arterial hydrogen ion concentration?
(A) Stimulation of central chemoreceptors by increased arterial PCO2
(B) Inhibition of the central chemoreceptors by increased plasma hydrogen ion concentration
(C) Stimulation of peripheral chemoreceptors by increased arterial PCO2
(D) Inhibition of the peripheral chemoreceptors by increased plasma hydrogen ion concentration
(E) Stimulation of the peripheral chemoreceptors by increased plasma hydrogen ion
concentration
161. A 5-month-old infant is admitted to the hospital for evaluation because of repeated episodes of
sleep apnea. During a ventilatory response test, his ventilation did not increase when PaCO2 was
increased, but decreased during hyperoxia. Which of the following is the most likely cause of
this infants apnea?
(A)
(B)
(C)
(D)
(E)

Bronchospasm
Diaphragmatic fatigue
Decreased irritant receptor sensitivity
Peripheral chemoreceptor hypersensitivity
Dysfunctional central chemoreceptors

Respiratory Physiology
162. A 14-year-old adolescent girl presents with a lump in the neck. Fine needle aspiration biopsy
reveals acinic cell carcinoma of the parotid gland. During the parotidectomy, there is
compression injury of the glossopharyngeal nerve. As a result, which of the following respiratory
reflexes will be impaired?
(A) Aortic chemoreceptor reflex
(B) Carotid body chemoreceptor reflex
(C) Hering-Breuer inflation reflex

(D) Irritant airway reflex


(E) Juxta pulmonary capillary (J) receptor
reflex

163. A 42-year-old woman with severe pulmonary fibrosis is evaluated by her physician and has the
following arterial blood gases: pH = 7.48, PaO2 = 55 mm Hg, and PaCO2 = 32 mm Hg. Which
statement best explains the observed value of PaCO2?
(A)
(B)
(C)
(D)
(E)

The increased pH stimulates breathing via peripheral chemoreceptors


The increased pH stimulates breathing via central chemoreceptors
The decreased PaO2 inhibits breathing via peripheral chemoreceptors
The decreased PaO2 stimulates breathing via peripheral chemoreceptors
The decreased PaO2 stimulates breathing via central chemoreceptors

164. The Hering-Breuer inflation reflex is mainly a protective mechanism that controls ventilation
under certain conditions. Which of the following best describes the effect of this reflex on
inspiration and expiration as well as the location of the stretch receptors that initiate the reflex?
(A)
(B)
(C)
(D)
(E)
(F)
(G)
(H)
(I)

Location of Stretch Receptors


Alveolar wall
Alveolar wall
Alveolar wall
Bronchi/bronchioles
Bronchi/bronchioles
Bronchi/bronchioles
Chest wall
Chest wall
Chest wall

Inspiration
No effect
Switches off
Switches on
No effect
Switches off
Switches on
No effect
Switches off
Switches on

Expiration
Switches off
No effect
Switches on
Switches off
No effect
Switches on
Switches off
No effect
Switches on

165. An increase in the activity of the afferent nerves associated with the Hering-Breuer reflex would
indicate:
(A)
(B)
(C)
(D)

Expiration is occurring
Inspiration is occurring
A decreasing blood volume with the pulmonary vasculature
Pulmonary congestion

Respiratory Physiology
166. The inherent rhythm for respiration appears to be located within the:
(A) Apneustic center
(B) Upper pons

(C) Lower pons


(D) Medulla

167. The basic rhythm of respiration is generated by neurons located in the medulla. Which of the
following limits the duration of inspiration and increases respiratory rate?
(A) Apneustic center
(B) Dorsal respiratory group
(C) Nucleus of the tractus solitarius
(D) Pneumotaxic center
(E) Ventral respiratory group
168. The pacemaker neurons responsible for respiratory rhythmogenesis are located in which of the
following regions of the brain?
(A)
(B)
(C)
(D)
(E)

Apneustic center in the pons


Central chemoreceptors in the medulla
Inspiratory neurons in the dorsal respiratory group
Pontine respiratory groups
Pre-Botzinger complex in the ventral respiratory group

169. When respiratory drive for increased pulmonary ventilation becomes greater than normal, a
special set of respiratory neurons that are inactive during normal quiet breathing then becomes
active, contributing to the respiratory drive. These neurons are located in which of the following
structures?
(A) Apneustic center
(B) Dorsal respiratory group
(C) Nucleus of the tractus solitaries

(D) Pneumotaxic center


(E) Ventral respiratory group

Respiratory Physiology
170. The following figure shows two ventilatory patterns: one normal and the other abnormal. Which
experimental maneuver listed below will create the abnormal pattern?

(A) Midpons transection with vagi intact


(B) Transection of the brain stem between the pons and medulla
(C) Midpons transection with vagi cut
(D) Transection rostral to the pons with vagi cut
(E) Transection rostral to the pons with vagi intact
171. An 18-year-old man is life-flighted to a Level 1 trauma center after being thrown from his
motorcycle. It is determined that he has a brain transection above the pons. How will this lesion
affect the control of breathing in this patient?
(A)
(B)
(C)
(D)
(E)

All breathing movements will cease.


The central chemoreceptors will no longer be able to exert any control over ventilation.
The peripheral chemoreceptors will no longer be able to exert any control over ventilation.
The Hering-Breuer reflex will be abolished.
The limbic system will no longer be able to exert any control over ventilation.

172. The following diagram shows the depth of respiration of a 45-year-old man who suffered a head
injury in an automobile accident. This crescendo-decrescendo pattern of breathing is called
which of the following?

(A) Apnea
(B) Biot breathing

(C) Cheyne-Stokes
breathing

(D) Hyperpnea
(E) Tachypnea

Respiratory Physiology
173. Cheyne-Stokes breathing is an abnormal breathing pattern characterized by a gradual increase in
the depth of breathing, followed by a progressive decrease in the depth of breathing that occurs
again and again about every minute, as shown in the following diagram. Which of the following
time points (V-Z) are associated with the highest Pco2 of lung blood and highest Pco2 of the
neurons in the respiratory center?

(A)
(B)
(C)
(D)
(E)

Lung Blood Respiratory Center


V
V
V
W
W
W
X
Z
Y
Z

For next 2 questions, use the table below to answer. Given the composition of the following gas
mixtures (balance nitrogen):
(A)
(B)
(C)
(D)

% O2 % CO2 % CO
21

0.1
95
5

20
2.5

20
2.5
0.1

174. Switching from breathing room air to which gas mixture for 2-3 minutes would produce the
greatest change in alveolar ventilation in a normal resting individual?

175. Switching from breathing room air to which gas mixture for 2-3 minutes would produce the least
change in alveolar ventilation in a normal resting individual?

Respiratory Physiology
Exercise/Unusual Environments
176. Which of the following changes occurs during strenuous exercise?
(A)
(B)
(C)
(D)
(E)

Ventilation rate and O2 consumption increase to the same extent


Systemic arterial PO2 decreases to about 70 mm Hg
Systemic arterial PCO2 increases to about 60 mm Hg
Systemic venous PCO2 decreases to about 20 mm Hg
Pulmonary blood flow decreases at the expense of systemic blood flow

177. In strenuous exercise, oxygen consumption and carbon dioxide formation can increase as much
as 20-fold. Alveolar ventilation increases almost exactly in step with the increase in oxygen
consumption. Which of the following best describes what happens to the mean arterial oxygen
tension (Po2), carbon dioxide tension (Pco2) and pH in a healthy athlete during strenuous
exercise?
(A)
(B)
(C)
(D)
(E)

Arterial Po2 Arterial Pco2 Arterial pH


Decreases
Decreases
Decreases
Decreases
Increases
Decreases
Increases
Decreases
Increases
Increases
Increases
Increases
No change
No change No change

178. During exercise a persons alveolar ventilation has doubled. This has been in response to
increased production of CO2 during the exercise and the blowing off all of this extra CO2. In
such a case what is the effect of doubling ventilation on the PaCO2?
(A) It will remain the same
(B) It will be doubled
(C) It will be quadrupled

(D) It will be halved


(E) It will be quartered

179. A healthy 32-year-old woman undergoes pulmonary exercise stress testing prior to starting a
training regimen in preparation for her first marathon. Normally, during moderate aerobic
exercise, which of the following occurs?
(A) PaO2 increases
(B) PaCO2 decreases
(C) Arterial pH decreases

(D) Alveolar ventilation increases


(E) Blood lactate level increases

Respiratory Physiology
180. A 26-year-old man training for a marathon reaches a workload that exceeds his anaerobic
threshold. If he continues running at or above this workload, which of the following will
increase?
(A) Alveolar ventilation
(B) Arterial pH
(C) PaCO2

(D) Plasma [HCO3]


(E) Firing of the central chemoreceptors

181. A 34-year-old medical student generates the flow-volume curves shown in the next diagram.
Curve W is a normal maximum expiratory flow-volume curve generated when the student was
healthy. Which of the following can best explain curve X?
(A)
(B)
(C)
(D)
(E)
(F)
(G)

Asthma attack
Aspiration of meat into the trachea
Heavy exercise
Light exercise
Normal breathing at rest
Pneumonia
Tuberculosis

182. Alveolar ventilation increases several-fold during strenuous exercise. Which of the following
factors is most likely to stimulate ventilation during strenuous exercise?
(A)
(B)
(C)
(D)
(E)

Collateral impulses from higher brain centers


Decreased mean arterial pH
Decreased mean arterial Po2
Decreased mean venous Po2
Increased mean arterial Pco2

183. Aerobic exercise causes which of the following changes in pulmonary physiology?
(A)
(B)
(C)
(D)
(E)

The overall V/Q ratio of the lungs decreases.


Diffusing capacity of the lungs increases.
Mean pulmonary artery pressure decreases.
Pulmonary blood flow decreases.
Pulmonary vascular resistance increases.

Respiratory Physiology
184. After living at an altitude of 3,500 meters for two months, a subject will have:
(A)
(B)
(C)
(D)

Higher than normal arterial PCO2


Elevated hematocrit
Abnormally low pressure in the pulmonary artery
Elevated erythropoietin

185. A 38-year-old woman moves with her family from New York City (sea level) to Leadville
Colorado (10,200 feet above sea level). Which of the following will occur as a result of residing
at high altitude?
(A)
(B)
(C)
(D)
(E)
(F)
(G)

Hypoventilation
Arterial PO2 greater than 100 mm Hg
Decreased 2,3-diphosphoglycerate (DPG) concentration
Shift to the right of the hemoglobin-O2 dissociation curve
Pulmonary vasodilation
Hypertrophy of the left ventricle
Respiratory acidosis

186. A group of third-year medical students accompanied a medical mission team to Peru. After
arriving at the airport in Bolivia, they hiked to a remote mountain village in the Andes at an
elevation of 18,000 ft. With a barometric pressure of 380 mm Hg at this altitude, what would be
the resulting PO2 of the dry inspired air?
(A) 160 mm Hg
(B) 100 mm Hg

(C) 80 mm Hg
(D) 70 mm Hg

(E) 38 mm Hg

187. A diver working at five times normal atmospheric pressure breathes a mixture of 50% helium
and 50% oxygen for four hours. Even before he begins to surface, he may encounter which of the
following problem(s)?
(A) Bends
(B) N2 narcosis

(C) Pneumothorax
(D) Oxygen toxicity

Respiratory Physiology
Ventilation-Perfusion
188. Which of the following is higher at the apex of the lung than at the base when a person is
upright?
(A) Ventilation
(B) Blood flow

(C) V/Q ratio


(D) PaCO2

(E) Lung compliance

189. A28-year-old woman on oral contraceptives develops tachypnea and reports dyspnea. A
ventilation/perfusion scan is ordered to check for pulmonary thromboemboli. Which of the
following best explains why, as she takes in a normal inspiration, more air goes to the alveoli at
the base of the lung than to the alveoli at the apex of the lung?
(A)
(B)
(C)
(D)
(E)

The alveoli at the base of the lung have more surfactant.


The alveoli at the base of the lung are more compliant.
The alveoli at the base of the lung have higher V/Q ratios.
There is a more negative intrapleural pressure at the base of the lung.
There is more blood flow to the base of the lung.

190. When a person is standing, blood flow in the lungs is


(A) Equal at the apex and the base
(B) Highest at the apex owing to the effects of gravity on arterial pressure
(C) Highest at the base because that is where the difference between arterial and venous pressure
is greatest
(D) Lowest at the base because that is where alveolar pressure is greater than arterial pressure
191. A healthy 10-year-old boy breathes quietly under resting conditions. His tidal volume is 400 ml
and ventilation frequency is 12/min. Which of the following best describes the ventilation of the
upper, middle, and lower lung zones in this boy?
(A)
(B)
(C)
(D)
(E)

Upper zone
Highest
Highest
Intermediate
Lowest
Same

Middle zone
Lowest
Intermediate
Lowest
Intermediate
Same

Lower zone
Intermediate
Lowest
Highest
Highest
Same

Respiratory Physiology
192. Compared with the apex of the lung, the base of the lung has
(A)
(B)
(C)
(D)

A higher pulmonary capillary PO2


A higher pulmonary capillary PCO2
A higher ventilation/perfusion (V/Q) ratio
The same V/Q ratio

193. Which of the following statements regarding the normal alveolar CO2 tension is true?
(A) It is equal in all alveoli
(B) It is highest at the base of vertical lungs
(C) It is directly proportional to the inspired O2 tension
(D) It is directly proportional to the alveolar ventilation
(E) It is equal to 46 mm Hg
194. Which of the following best shows the apex vs. the base of the lung in a standing subject (>
means greater than; < means less than)?

(A)
(B)
(C)
(D)
(E)

Ventilation(V)
Perfusion(Q)
(ml/min per unit volume) (ml/min per unit volume)
Apex > Base
Apex > Base
Apex > Base
Apex > Base
Apex > Base
Apex < Base
Apex < Base
Apex < Base
Apex < Base
Apex < Base

V/Q
Apex > Base
Apex < Base
Apex < Base
Apex > Base
Apex < Base

195. If alveolus X has a ventilation-perfusion ratio of 0.85 and alveolus Y has a ventilation-perfusion
ratio of 0.65, which alveolus has the higher PO2?
(A) X

(B) Y

196. If alveolus X has a ventilation-perfusion ratio of 0.85 and alveolus Y has a ventilation-perfusion
ratio of 0.65, which alveolus has the higher PCO2?
(A) X

(B) Y

197. If alveolus X has a ventilation-perfusion ratio of 0.85 and alveolus Y has a ventilation-perfusion
ratio of 0.65, capillary blood leaving which alveolus will have the greatest O2 content?
(A) X

(B) Y

Respiratory Physiology
198. If alveolus X has a ventilation-perfusion ratio of 0.85 and alveolus Y has a ventilation-perfusion
ratio of 0.65, capillary blood leaving which alveolus will have the greatest CO2 content?
(A) X

(B) Y

199. If alveolus X has a ventilation-perfusion ratio of 0.85 and alveolus Y has a ventilation-perfusion
ratio of 0.65, capillary blood leaving which alveolus will have the highest pH?
(A) X

(B) Y

200. Which of the following statements best characterizes the pattern of ventilation in the lungs
during quiet breathing?
(A) Surfactant keeps each region of the lung equally distended and ventilated.
(B) Gravity in the erect individual keeps the base of the lung more poorly expanded and it
receives less ventilation than the apex.
(C) Gravity in the erect individual keeps the base of the lung more poorly expanded but it
receives more ventilation than the apex.
(D) Gravity in the erect individual keeps the base of the lung more expanded and ventilated than
the apex.
(E) Gravity in the erect individual keeps the base of the lung more expanded and less ventilated
than the apex.
201. Going from a quiet, standing position to climbing a set of stairs, which of the following
conditions will be present?
(A)
(B)
(C)
(D)
(E)
(F)

Apical Flow Basal Flow

Respiratory Physiology
202. During exercise, the oxygenation of blood is increased
not only by increased alveolar
ventilation but also by a greater diffusing capacity of the respiratory membrane for transporting
oxygen into the blood. Which of the following sets of changes occur during exercise?

(A)
(B)
(C)
(D)
(E)

Surface Area of
Ventilation-Perfusion
Respiratory Membrane
Ratio
Decrease
Improvement
Increase
Improvement
Increase
No change
No change
Improvement
No change
No change

203. A 23-year-old medical student has mixed venous oxygen and carbon dioxide tensions of 40 mm
Hg and 45 mm Hg, respectively. A group of alveoli are not ventilated in this student because
mucus blocks a local airway. What are the alveolar oxygen and carbon dioxide tensions distal to
the mucus block (in mm Hg)?
(A)
(B)
(C)
(D)
(E)

Carbon Dioxide Oxygen


40
100
40
40
45
40
50
50
90
40

204. A child has been eating round candies approximately 1 and 1.5 cm in diameter and inhaled one
down his airway blocking his left bronchiole. Which of the following will describe the changes
that occur?
(A)
(B)
(C)
(D)
(E)

Left Lung Alveolar Pco2 Left Lung Alveolar Po2 Systemic Arterial Po2

205. A young child inhales a marble that lodges in the airway supplying the lower lobe of the right
lung, completely blocking the ventilation of this part of the lung. Which one of the following
would most likely increase as a result?
(A) Anatomical dead space
(B) Anatomical shunt flow
(C) Azygos vein flow

(D) Physiological dead space


(E) Physiological shunt flow

Respiratory Physiology
206. A 45-year-old man at sea level has an inspired oxygen tension of 149 mm Hg, nitrogen tension of
563 mm Hg, and water vapor pressure of 47 mm Hg. A small tumor pushes against a pulmonary
blood vessel that completely blocks the blood flow to a small group of alveoli. What are the
oxygen and carbon dioxide tensions of the alveoli that are not perfused (in mm Hg)?
(A)
(B)
(C)
(D)
(E)

Carbon Dioxide Oxygen


0
0
0
149
40
104
47
149
45
149

207. An elderly male patient with thrombophlebitis in one leg develops a pulmonary embolus that
completely blocks blood flow to the upper lobe of his left lung. Which one of the following
would most likely increase as a result?
(A) Functional residual capacity (FRC)
(B) Physiological dead space
(C) FEV1/FVC ratio

(D) Physiological shunt flow


(E) Residual volume (RV)

208. The next diagram shows two lung units (S and T) with their blood supplies. Lung unit S has an
ideal relationship between blood flow and ventilation. Lung unit T has a comprised blood flow.
What is the relationship between alveolar dead space (DALV), physiologic dead space (DPHY) and
anatomic dead space (DANAT) for these lung units?

(A)
(B)
(C)
(D)
(E)

Lung unit S
DPHY < DANAT
DPHY = DALV
DPHY = DANAT
DPHY = DANAT
DPHY > DANAT

Lung unit T
DPHY = DANAT
DPHY > DALV
DPHY < DANAT
DPHY > DANAT
DPHY < DANAT

Respiratory Physiology
209. The O2-CO2 diagram here shows a ventilation-perfusion ratio line for the normal lung. Which of
the following best describes the effect of decreasing ventilation-perfusion ratio on the alveolar
Po2 and Pco2?

(A)
(B)
(C)
(D)
(E)

Carbon Dioxide
Tension
Decrease
Decrease
Decrease
Increase
Increase

Oxygen
Tension
Decrease
Increase
No change
Decrease
Increase

Questions 210 and 211

210. A 67-year-old man has a solid tumor that pushes against an airway partially obstructing air flow
to the distal alveoli. Which point on the ventilation-perfusion line of the O2-CO2 diagram
corresponds to the alveolar gas of these distal alveoli?

211. A 55-year-old male has a pulmonary embolism that partially blocks the blood flow to his right
lung. Which point on the ventilation-perfusion line of the O2-CO2 diagram corresponds to the
alveolar gas of his right lung?

Respiratory Physiology
212. A 59-year-old man with right lower lobar pneumonia begins experiencing increasing respiratory
distress. The patient is moved to the ICU, intubated, and placed on a mechanical ventilator. With
the patient positioned on his left side, which of the following variables will be lower in the left
lung compared to the right lung?
(A) Alveolar ventilation per unit volume
(B) Lung compliance
(C) PaCO2

(D) Pulmonary blood flow


(E) V/Q ratio

213. When the ventilation-perfusion ratio of a lung unit deceases, the alveoli in that unit develop a:
(A) Higher PO2
(B) Lower PN2
(C) Higher PO2 and lower PCO2

(D) Higher PCO2


(E) Higher PN2 and higher PO2

214. A hospitalized patient with respiratory disease is found to have some lung regions with very low
ventilation/perfusion (VA/Q) ratios. Compared to pulmonary venous blood coming from regions
of the lung with normal VA/Q ratios, the pulmonary venous blood draining these low VA/Q
regions would most likely have:
(A) A higher PO2 and a higher PCO2
(B) A higher PO2 and a lower PCO2
(C) A higher PO2 and a normal PCO2

(D) A lower PO2 and a higher PCO2


(E) A lower PO2 and a lower PCO2

Pulmonary Vascular System


215. A 67-year-old man is admitted as an emergency to University Hospital because of severe chest
pain. A Swan-Ganz catheter is floated into the pulmonary artery, the balloon is inflated, and the
pulmonary wedge pressure is measured. The pulmonary wedge pressure is used clinically to
monitor which of the following pressures?
(A) Left atrial pressure
(B) Left ventricular pressure
(C) Pulmonary artery diastolic pressure

(D) Pulmonary artery systolic pressure


(E) Pulmonary capillary pressure

Respiratory Physiology
216. A 30-year-old woman performs a valsalva maneuver about 30 min after eating lunch. Which of
the following best describes the changes in pulmonary and systemic blood volumes that occur in
this woman?
(A)
(B)
(C)
(D)
(E)
(F)
(G)
(H)
(I)

Pulmonary Volume Systemic Volume


Decreases
Decreases
Decreases
Increases
Decreases
No change
Increases
Decreases
Increases
Increases
Increases
No change
No change
Decreases
No change
Increases
No change
No change

217. Compared with the systemic circulation, the pulmonary circulation has a
(A) Higher blood flow
(B) Lower resistance
(C) Higher arterial pressure

(D) Higher capillary pressure


(E) Higher cardiac output

218. Which of the following sets of differences best describes the hemodynamics of the pulmonary
circulation when compared to the system circulation?
(A)
(B)
(C)
(D)
(E)
(F)

Flow Resistance Arterial pressure


Higher
Higher
Higher
Higher
Lower
Lower
Lower
Higher
Lower
Lower
Lower
Lower
Same
Higher
Lower
Same
Lower
Lower

219. Which of the following will decrease pulmonary blood flow resistance?
(A) IV injection of norepinephrine
(B) Inhalation to total lung capacity

(C) Breathing 5% O2
(D) Having the lung at FRC

220. In which vascular bed does hypoxia cause vasoconstriction?


(A) Coronary
(B) Pulmonary

(C) Cerebral
(D) Muscle

(E) Skin

Respiratory Physiology
221. A 32-year-old man drives to the top of Pikes Peak where the oxygen tension is 85 mm Hg.
Which of the following best describes the effects of a hypoxic environment on the pulmonary
and systemic vascular resistances?
(A)
(B)
(C)
(D)
(E)
(F)
(G)
(H)
(I)

Pulmonary Vascular Resistance Systemic Vascular Resistance


Decreases
Decreases
Decreases
Increases
Decreases
No change
Increases
Decreases
Increases
Increases
Increases
No change
No change
Decreases
No change
Increases
No change
No change

222. A 65-year-old man with emphysema due to 34 years of cigarette smoking is admitted to hospital
due to dyspnea. With further tests the mean pulmonary arterial pressure is determined to be 45
mm Hg at rest. He is hypoxic (Po2 = 49 mm Hg), hypercapnic (85 mm Hg), and slightly acidotic.
The cardiovascular and oxygen changes are due to which of the following?
(A) Increased arterial Pco2
(B) Increased parasympathetic activity
(C) Decreased alveolar Po2

(D) Decreased pH
(E) Decreased pulmonary resistance

223. Noninvasive color Doppler ultrasound studies are ordered on a term infant and a preterm infant
of 28-weeks gestation. Which of the following is likely to have a lower value in the preterm
infant compared to the term infant?
(A)
(B)
(C)
(D)
(E)

Pulmonary artery pressure


Pulmonary blood flow
Pulmonary vascular resistance
Pulmonary capillary hydrostatic pressure
Blood flow from the pulmonary artery through the ductus arteriosus

224. A 42-week gestation infant is delivered by cesarean section. Which of the following occurs with
the babys first diaphragmatic respiration?
(A)
(B)
(C)
(D)
(E)

PaO2 increases
Pulmonary vascular resistance increases
Pulmonary capillary hydrostatic pressure increases
Systemic vascular resistance decreases
All of the fetal vascular channels functionally close

Respiratory Physiology
225. A32-year-old man is hospitalized with severe respiratory disease following aspiration pneumonia.
Inhaled nitric oxide is administered and he is placed in a prone position to improve oxygenation.
Values obtained after the administration of nitric oxide are:
Mean pulmonary capillary oxygen content
Arterial oxygen content
Mixed venous oxygen content
Cardiac output

19 mL/dL
18 mL/dL
14 mL/dL
6 L/min

Which of the following is the patients shunt fraction (the ratio of shunted to total pulmonary
blood flow)?
(A) 10%

(B) 20%

(C) 30%

(D) 40%

(E) 50%

226. A32-year-old man is hospitalized with severe respiratory disease following aspiration pneumonia.
Inhaled nitric oxide is administered and he is placed in a prone position to improve oxygenation.
Values obtained after the administration of nitric oxide are: What is the patients oxygen
consumption?
Mean pulmonary capillary oxygen content
Arterial oxygen content
Mixed venous oxygen content
Cardiac output
(A) 200 mL/min
(B) 210 mL/min

19 mL/dL
18 mL/dL
14 mL/dL
6 L/min

(C) 220 mL/min


(D) 230 mL/min

227. Pulmonary edema in CHF is promoted by which of the following?


(A)
(B)
(C)
(D)
(E)

Decreased pulmonary interstitial oncotic pressure


Increased pulmonary capillary hydrostatic pressure
Increased pulmonary capillary oncotic pressure
Increased pulmonary interstitial hydrostatic pressure
Decreased pulmonary capillary permeability

(E) 240 mL/min

Respiratory Physiology
228. A patient with a myocardial infarction develops progressive dyspnea and hypoxemia suggestive
of cardiogenic shock. She is transferred to the medical intensive care unit and a Swan-Ganz
catheter is inserted. The patient is found to have a pulmonary artery wedge pressure of 30 mm
Hg, which is indicative of a decrease in which of the following?
(A)
(B)
(C)
(D)
(E)

Left atrial pressure


Left ventricular end-diastolic pressure
Left ventricular preload
Net fluid absorption into the pulmonary capillaries
Pulmonary capillary hydrostatic pressure

229. A 65-year-old smoker develops a squamous cell bronchogenic carcinoma, which metastasizes to
the tracheobronchial and parasternal lymph nodes. The chest x-ray is consistent with
accumulation of fluid in the pulmonary interstitial space. Flow of fluid through the lymphatic
vessels will be decreased if there is an increase in which of the following?
(A) Capillary pressure
(B) Capillary permeability
(C) Interstitial protein concentration

(D) Capillary oncotic pressure


(E) Central venous pressure

230. A 19-year-old man suffers a full-thickness burn over 60% of his body surface area. A systemic
Pseudomonas aeruginosa infection occurs and severe pulmonary edema follows 7 days later.
Data collected from the patient follow: plasma colloid osmotic pressure, 19 mm Hg; pulmonary
capillary hydrostatic pressure, 7 mm Hg; and interstitial fluid hydrostatic pressure, 1 mm Hg.
Which of the following sets of changes has occurred in the lungs of this patient as a result of the
burn and subsequent infection?

(A)
(B)
(C)
(D)
(E)

Lymph
Plasma Colloid Pulmonary Capillary
Flow
Osmotic pressure
Permeability
Decrease
Decrease
Decrease
Increase
Decrease
Decrease
Increase
Decrease
Increase
Increase
Increase
Decrease
Increase
Increase
Increase

Respiratory Physiology
231. A 67-year-old man who is a candidate for cardiac transplantation undergoes cardiac
catheterization to assess his hemodynamic status. Findings include:
Pulmonary artery pressure = 35 mm Hg
Left atrial pressure = 15 mm Hg

Cardiac output = 4 L/min


Right atrial pressure=10 mm Hg

Which of the following values is his pulmonary vascular resistance?


(A) 0.16 L/min/mm Hg
(B) 0.2 L/min/mmHg

(C) 5 mm Hg/L/min
(D) 6.25 mm Hg/L/min

232. Which of the following conditions will cause a decrease in pulmonary vascular resistance?
(A)
(B)
(C)
(D)
(E)

Alveolar hypoxia
Decreased pH in the pulmonary artery
Increased cardiac output
Inflation of the lungs to total lung capacity
Sympathetic stimulation of the pulmonary vessels

233. Which diagram best illustrates the pulmonary vasculature when the cardiac output has increased
to a maximum extent?

Respiratory Physiology
234. Arterial Po2 is 100 mm Hg and arterial Pco2 is 40 mm Hg. Total blood flow to all muscle is 700
ml/min. There is a sympathetic activation resulting in a decrease in blood flow to 350 ml/min.
Which of the following will occur?
(A)
(B)
(C)
(D)
(E)
(F)
(G)

Venous Po2 Venous Pco2

Immunology & Respiratory System


235. Which of the following is the major route for removal of small particles from the alveoli?
(A) Bulk flow
(B) Diffusion

(C) Expectoration
(D) Phagocytosis

(E) Ciliary transport

Pathophysiology
236. What changes in lung function occur as a result of a (right) pneumothorax?
(A)
(B)
(C)
(D)
(E)

The intrapleural pressure in the affected area equals to atmospheric pressure.


The chest wall on the affected side recoils inward.
There is hyperinflation of the affected lung.
The V/Q ratio on the affected side increases above normal.
The mediastinum shifts further to the right with each inspiration.

237. A 34-year-old male sustains a bullet wound to the chest that causes a pneumothorax. Which of
the following best describes the changes in lung volume and thoracic volume in this man,
compared to normal?
(A)
(B)
(C)
(D)
(E)
(F)

Lung volume Thoracic volume


Decreased
Decreased
Decreased
Increased
Decreased
No change
Increased
Decreased
Increased
Increased
No change
Decreased

Respiratory Physiology
238. A 29-year-old woman is admitted to the hospital because of increasing dyspnea and swelling of
both feet. An examination of her chest shows a severe pectus excavatum with only 2 cm of space
between the vertebral bodies and the sternum. Pulmonary function tests show FVC and
FEV1/FVC values that were 15% and 100%, respectively, of predicted. Which of the following
laboratory measurements will most likely be below normal in this patient?
(A) Elastic recoil of the chest wall
(B) Arterial PCO2
(C) Hemoglobin concentration

(D) Plasma bicarbonate concentration


(E) Arterial pH

239. Which of the following pulmonary function test results is consistent with a diagnosis of allergic
bronchospasm?
(A) An increased forced vital capacity
(B) A decreased FEV1/FVC
(C) An increased diffusing capacity

(D) A decreased residual volume


(E) An increased lung compliance

240. A 12-year-old boy has a severe asthmatic attack with wheezing. He experiences rapid breathing
and becomes cyanotic. His arterial PO2 is 60 mmHg and his PCO2 is 30 mmHg. Which of the
following statements about this patient is most likely to be true?
(A)
(B)
(C)
(D)
(E)

Forced expiratory volume/forced vital capacity (FEV1 /FVC) is increased


Ventilation/perfusion (V/Q) ratio is increased in the affected areas of his lungs
His arterial PCO2 is higher than normal because of inadequate gas exchange
His arterial PCO2 is lower than normal because hypoxemia is causing him to hyperventilate
His residual volume (RV) is decreased

Respiratory Physiology
241. A 10-year-old girl presents in the pediatricians office with a nonproductive cough, wheezing,
and shortness of breath. Her mother reports that these symptoms have been episodic over the past
two years, but this time her daughter has been getting worse over the past several days. Using a
screening pulmonary function machine in the office, the girl first generated curve #1. Subsequent
to administration of an inhaled bronchodilator, she generated curve #2. What is the most likely
explanation for the increased flow measured in curve #2?
(A) Increased patient effort
(B) Mast cell degranulation in bronchial
smooth muscle
(C) Activation of -adrenergic receptors in
bronchial smooth muscle
(D) Increased
parasympathetic
nerve
discharge to bronchial smooth muscle
(E) Stimulation of 2-adrenergic receptors
in bronchial smooth muscle

242. An 18-year-old woman with a 9-year history of wheezing on exertion is referred for pulmonary
function tests. The figure below represents the spirometry tracing of a forced vital capacity. Her
total lung capacity was 130% of predicted. Which of the following values will most likely be
above normal?

(A) Vital capacity


(B) Residual volume
(C) Expiratory reserve volume

(D) FEV1/FVC
(E) Maximum voluntary ventilation

Respiratory Physiology
243. A 66-year-old male with a long history of cigarette smoking complains of difficulty breathing
and a persistent cough productive of thick mucus. Pulmonary function tests on this patient reveal
increased total lung capacity (TLC) and residual volume (RV), a normal vital capacity (FVC), a
reduced FEV1, and a FEV1/FVC ratio of 0.58. This spirometric pattern is most consistent with
which of the following?
(A) A decreased pulmonary diffusion capacity
(B) An obstructive lung disease only
(C) A combination of restrictive and obstructive lung disease
(D) A restrictive lung disease only
(E) A normal pattern with no evidence of either restrictive or obstructive lung disease
244. The diagram below shows a forced expiration for a healthy person (curve X) and a person with a
pulmonary disease (curve Z). What is the FEV1/FVC ratio (as a percent) in these individuals?

(A)
(B)
(C)
(D)
(E)
(F)

Person X Person Z
80
50
80
40
100
80
100
60
90
50
90
60

245. The following diagram shows forced expirations from a person with healthy lungs (curve X) and
from a patient (curve Z). Which of the following can best explain the results from the patient?
(A)
(B)
(C)
(D)
(E)
(F)
(G)

Asbestosis
Emphysema
Fibrotic pleurisy
Pleural effusion
Pneumothorax
Silicosis
Tuberculosis

Respiratory Physiology
246. The maximum expiratory flow-volume curves shown in the next diagram were obtained from a
healthy individual (red curve) and a 57-year old man who complains of shortness of breath
(green curve). Which of the following disorders is most likely present in the man?
(A)
(B)
(C)
(D)
(E)
(F)

Asbestosis
Emphysema
Kyphosis
Scoliosis
Silicosis
Tuberculosis

247. The maximum expiratory flow-volume curve shown in the next diagram (red line) was obtained
from a 75-year-old man who smoked 40 cigarettes per day for the past 60 years. The green
flow-volume curve was obtained from the man during resting conditions. Which of the following
sets of changes are most likely to apply to this man?

(A)
(B)
(C)
(D)
(E)

Exercise tolerance Total lung capacity Residual volume


Decreased
Decreased
Decreased
Decreased
Increased
Increased
Decreased
Normal
Normal
Increased
Increased
Increased
Normal
Decreased
Decreased

Respiratory Physiology
248. In the maximal expiratory flow-volume curves below, curve A would be typical of which of the
following clinical presentations?

(A) 75-year-old man who has smoked two packs of cigarettes per day for 60 years. His breath
sounds are decreased bilaterally, and his chest x-ray shows flattening of the diaphragm.
(B) A 68-year-old man presents with a dry cough that has persisted for 3 months. His chest x-ray
shows opacities in the lower and middle lung fields. The man states he was exposed to
asbestos for approximately 10 years when he worked in a factory in his thirties.
(C) A 57-year-old woman with pulmonary fibrosis who presents to the emergency room with
shortness of breath.
(D) An 84-year-old woman with a history of myocardial infarction who reports shortness of
breath that worsens in the recumbent position.
(E) A healthy 22-year-old man getting his Army enlistment physical. He has never smoked, but
is tired that morning, and does not use much effort while exhaling.
249. A 78-year old man who smoked 60 cigarettes per day for 55 years complains of shortness of
breath. The patient is diagnosed with chronic pulmonary emphysema. Which of the following
sets of changes is present in this man, compared to a healthy, nonsmoker?
(A)
(B)
(C)
(D)
(E)
(F)

Pulmonary Compliance Lung Elastic recoil Total Lung Capacity


Decreased
Decreased
Decreased
Decreased
Decreased
Increased
Decreased
Increased
Increased
Increased
Decreased
Decreased
Increased
Decreased
Increased
Increased
Increased
Increased

Respiratory Physiology
250. The volume-pressure curves shown here were obtained from a normal subject and a patient
suffering from a pulmonary disease. Which of the following abnormalities is most likely present
in the patient?
(A)
(B)
(C)
(D)
(E)
(F)

Asbestosis
Emphysema
Mitral obstruction
Rheumatic heart disease
Silicosis
Tuberculosis

251. Which of the following decreases with emphysema?


(A) Alveolar Pco2
(B) Cardiac output

(C) Diffusion area


(D) Pulmonary artery pressure

252. A76-year-old patient with emphysema presents for his annual pulmonary function testing to
assess the progression of his disease. As a result of alveolar septal departitioning in emphysema,
there is a decrease in which of the following?
(A) Airway resistance
(B) Alveolar dead space

(C) Diffusing capacity


(D) Lung compliance

(E) Total lung capacity

Respiratory Physiology
253. A 68-year-old man with chronic obstructive pulmonary disease entered the emergency
department complaining of shortness of breath. His respirations were 35 per minute and labored.
He had a productive cough and rales were heard over all lung fields. The patient had a rather
ashen complexion and his nail beds gave clear evidence of cyanosis. An arterial blood sample
was obtained and a chest x-ray was ordered. The patient was then placed on an O2 mask
delivering 40% O2. One-half hour later, the physician was called to the bedside by the nurse who
found the patient unresponsive. The patients complexion had changed to a flushed pink with no
trace of cyanosis. His respirations were quiet at a rate of 6 per minute and a tidal volume of 300
mL. Repeat arterial blood gases showed that his arterial PCO2 had increased from 55 to 70 mm
Hg, and his PaO2 increased from 55 to 70 mm Hg. Oxygen therapy most likely resulted in which
of the following?
(A)
(B)
(C)
(D)
(E)

Alveolar hypoventilation
Hypoxic pulmonary vasoconstriction
Increased firing of carotid body chemoreceptors
Elimination of the hypercapnic drive
Oxygen toxicity

254. A patient with restrictive lung disease (RLD) typically has


(A) An increased forced expiratory volume in 1 second (FEV1) and a normal lung compliance
(B) A decreased FEV1 and an increased lung compliance
(C) A decreased FEV1 and a decreased lung compliance
(D) An increased FEV1 and an increased lung compliance
(E) An increased FEV1 and a decreased lung compliance
255. The next diagram shows forced expirations from a person with healthy lungs (curve X) and from
a patient (curve Z). Which of the following is most likely present in the patient?
(A)
(B)
(C)
(D)
(E)

Asthma
Bronchospasm
Emphysema
Old age
Silicosis

Respiratory Physiology
256. Which of the following graphs and data set best represents a patient with restrictive airway
disease?
Data Set A:
Data Set B:

FEV1 low
FEV1 high

(A) Graph 1 and Data Set A


(B) Graph 1 and Data Set B
(C) Graph 2 and Data Set B
(D) Graph 3 and Data Set A
(E) Graph 3 and Data Set B

257. A 75-year-old man worked for 5 years in a factory in his early 40s where asbestos was used as an
insulator. The man is diagnosed with asbestosis. Which of the following sets of changes is
present in this man, compared to a person with healthy lungs?
(A)
(B)
(C)
(D)
(E)
(F)

Pulmonary compliance Lung elastic recoil Total lung capacity


Decreased
Decreased
Decreased
Decreased
Increased
Increased
Decreased
Increased
Decreased
Increased
Decreased
Decreased
Increased
Decreased
Increased
Increased
Increased
Increased

258. The volume-pressure curves shown in the next diagram were obtained from a young, healthy
subject and a patient. Which of the following best describes the condition of the patient?
(A)
(B)
(C)
(D)
(E)

Asthma
Bronchospasm
Emphysema
Old age
Silicosis

Respiratory Physiology
259. Compared to a normal healthy person, how do total lung capacity and maximal expiratory flow
patient change with restrictive lung disease?
(A)
(B)
(C)
(D)

Total Lung Capacity Maximum Expiratory Flow

260. A 54-year-old man with severe asbestosis reports worsening of his dyspnea. Pulmonary function
tests are ordered and the patient is instructed to take in a maximal inspiration and then to exhale
as hard and fast as he can to generate a maximal expiratory flow-volume (MEFV) curve. As a
result, the patient generates curve C shown below:

(A)
(B)
(C)
(D)
(E)

FVC
FVC
FEV1
FEV1
(liters) (% predicted) (liters) (% predicted) FEV1/FVC
4.2
85
3.4
90
81
4.2
85
2.1
32
50
3.1
48
2.8
50
90
3.1
48
2.0
40
65
1.5
25
1.0
20
67

Respiratory Physiology
261. A 62-year-old man complains to his physician that he has difficulty breathing. The following
diagram shows a maximum expiratory flow-volume (MEFV) curve from the patient (green line)
and from a typical healthy individual (red curve). Which of the following best explains the
MEFV curve of the patient?
(A)
(B)
(C)
(D)
(E)

Asbestosis
Asthma
Bronchospasm
Emphysema
Old age

262. A 21-year-old woman presents with cough and shortness of breath. The physician conducts a
pulmonary function screening test in his office, and the patient generates the maximum
flow-volume curve shown to the right of the normal curve in the diagram below. These findings
are consistent with which of the following conditions?

(A) Asthma
(B) Chronic bronchitis
(C) Cystic fibrosis

(D) Decreased effort


(E) Sarcoidosis

263. Which of the following laboratory values is consistent with pulmonary fibrosis?
(A)
(B)
(C)
(D)
(E)

Decreased diffusing capacity of the lung


Increased residual volume
Decreased FEV1/FVC
Increased lung compliance
Increased airway resistance corrected for lung volume

Respiratory Physiology
264. A 49-year-old coal miner presents with dyspnea, a nonproductive cough, and decreased exercise
tolerance. Lung function tests reveal: total lung capacity = 3.34 L (56% of predicted), residual
volume = 0.88 L (54% of predicted), and forced vital capacity = 1.38 L (30% of predicted). His
arterial PO2 is 68 mm Hg. Which of the following values will be approximately normal?
(A) Diffusing capacity
(B) FEV1/FVC ratio
(C) Functional residual capacity

(D) Lung compliance


(E) V/Q ratio

265. A 56-year-old woman presents to her physician complaining of fatigue, headaches, and dyspnea
on exertion. She states that she sometimes gets blue lips and fingers when she tries to exercise.
Pulmonary function tests reveal an increase, rather than a decrease, in the diffusing capacity of
the lung. Which of the following conditions best accounts for an increase in the diffusing
capacity?
(A) Congestive heart failure
(B) COPD
(C) Fibrotic lung disease

(D) Polycythemia
(E) Pulmonary embolism

266. Ventilation-perfusion (V/Q) abnormalities occurring in a patient with lobar pneumonia will
generally cause a decrease in which of the following?
(A) Alveolar ventilation
(B) Arterial carbon dioxide tension
(C) Arterial pH

(D) Anion gap


(E) A-a gradient for oxygen

267. If an area of the lung is not ventilated because of bronchial obstruction, the pulmonary capillary
blood serving that area will have a PO2 that is
(A) Equal to atmospheric PO2
(B) Equal to mixed venous PO2
(C) Equal to normal systemic arterial PO2

(D) Higher than inspired PO2


(E) Lower than mixed venous PO2

268. A 49-year-old man has a pulmonary embolism that completely blocks blood flow to his left lung.
As a result, which of the following will occur?
(A)
(B)
(C)
(D)
(E)

Ventilation/perfusion (V/Q) ratio in the left lung will be zero


Systemic arterial PO2 will be elevated
V/Q ratio in the left lung will be lower than in the right lung
Alveolar PO2 in the left lung will be approximately equal to the PO2 in inspired air
Alveolar PO2 in the right lung will be approximately equal to the PO2 in venous blood

Respiratory Physiology
269. Which of the following occurs if the blood flow to an alveolus is totally obstructed by a
pulmonary thromboembolism?
(A)
(B)
(C)
(D)
(E)

The V/Q ratio of the alveolus equals zero


The PO2 of the alveolus will be equal to that in the inspired air
The PO2 of the alveolus will be equal to the mixed venous PO2
There will be an increase in shunting (venous admixture) in the lung
There will be a decrease in alveolar dead space

270. A person with a ventilation/perfusion (V/Q) defect has hypoxemia and is treated with
supplemental O2. The supplemental O2 will be most helpful if the persons predominant V/Q
defect is
(A) Dead space
(B) Shunt

(C) High V/Q


(D) Low V/Q

(E) V/Q = 0
(F) V/Q =

271. Which of the following will be greater than normal in a patient with a low V/Q ratio?
(A) PaCO2
(B) PaO2
(C) A-a gradient

(D) Oxygen dissolved in blood


(E) Oxygen combined with hemoglobin

272. A 57-year-old man undergoes total knee replacement for severe degenerative joint disease. Four
days after surgery he develops an acute onset of shortness of breath and right-sided pleuritic
chest pain. He is now in moderate distress with a respiratory rate of 28 breaths/min, tidal volume
of 450 mL, heart rate of 120 bpm, and blood pressure of 125/85 mm Hg. Arterial blood gases on
room air at a barometric pressure = 760 mm Hg and R = 0.8 were PaO2 = 60 mm Hg, SaO2 = 90%,
PaCO2 = 30 mm Hg, pH = 7.50, [HCO3] = 22 mEq/L, and PECO2 = 10 mm Hg. The right lower
extremity is healing well, but is red, tender, warm to touch, and has 2+ pitting edema. The most
likely cause of these postoperative findings is:
(A) Atelectasis
(B) Pneumonia
(C) Pneumothorax

(D) Pulmonary embolism


(E) Sepsis

Respiratory Physiology
273. A 43-year-old woman develops shortness of
breath following a cholecystectomy.
Bronchial breath sounds and crackles are
heard over all lung fields and the lungs are
dull on percussion. A chest x-ray
demonstrates a pattern of diffuse
opacification characteristic of atelectasis.
Intrapulmonary shunting will cause which of
the following changes in arterial blood gas
values?

(A)
(B)
(C)
(D)

pH PaCO2 PaO2

Hypoxia
274. A reduction of arterial O2 tension is typical of which one of the following?
(A) Anemia
(B) CO poisoning
(C) Moderate exercise

(D) Cyanide poisoning


(E) Hypoventilation

275. Which of the following clinical or laboratory findings best reveals the presence of a hypoxic
ventilatory response in a patient with arterial hypoxemia?
(A) Increased anion gap
(B) Decreased arterial PCO2
(C) Increased plasma bicarbonate

(D) Decreased plasma pH


(E) Increased A-a PO2 difference

276. A 78-year-old male presents to the emergency room complaining of dyspnea. Arterial blood gas
results, obtained with the patient breathing room air (barometric pressure = 760 mm Hg), are as
follows: arterial PO2, 55 mm Hg; arterial saturation, 78%; PaCO2, 69 mm Hg; pH, 7.17.
Administration of supplemental oxygen via a nasal cannula increased the arterial PO2 to 125 mm
Hg. Based on these results, which of the following is most likely the primary cause of this
patients hypoxemia?
(A) Ambient hypoxia
(B) Alveolar hypoventilation
(C) Diffusion abnormality

(D) Ventilation-perfusion mismatch


(E) Increased physiological shunt

Respiratory Physiology
277. A 30-year-old woman is admitted to the emergency department with dyspnea, tachycardia,
confusion, and other signs of hypoxia. The following laboratory data were obtained while the
patient was breathing room air:
PaO2 = 67 mm Hg
PaCO2 = 60 mm Hg
pH = 7.27
[HCO3] = 26mEq/L
[Hb] = 15 g%

SaO2 =90%
PvO2 = 30 mm Hg
SvO2 = 55%
VO2 = 350 mL/min
CaO2-CvO2 = 7 mL O2/dL

Which of the following is the most appropriate classification of the patients hypoxia?
(A) Hypoxic hypoxia (hypoxemia)
(B) Anemic hypoxia
(C) Stagnant (hypoperfusion) hypoxia

(D) Histotoxic hypoxia


(E) Carbon monoxide poisoning

278. The following diagram shows a lung with a large shunt in which mixed venous bypasses the
oxygen exchange areas of the lung. Breathing room air produces the oxygen partial pressures
shown on the diagram. What is the oxygen tension of the arterial blood (in mm Hg) when the
person breathes 100% oxygen and the inspired oxygen tension is over 600 mm Hg?

(A) 40

(B) 55

(C) 60

(D) 175

(E) 200

(F) 400

(G) 600

Respiratory Physiology
279. A 68-year-old woman convalescing from surgery developed a fever, hypoxemia, and shortness of
breath. She was given 100% O2 for 30 minutes, and the laboratory results were as follows:
PaO2 = 95 mm Hg [HCO3] =22mEq/L
PaCO2 = 33 mm Hg [Hb] = 15 g%
pH = 7.46
SaO2 = 95%
The response to 100% O2 reveals that the patient has which of the following?
(A) Alveolar hypoventilation
(B) Diffusion impairment
(C) V/Q inequality with low V/Q units

(D) Right-to-left shunting


(E) Carbon monoxide poisoning

280. A 36-year-old woman is found comatose at her home and is life-flighted to the nearest regional
medical center. Blood gases reveal a normal PaO2 but a lower-than-normal arterial O2 saturation.
Which of the following conditions is most consistent with the findings?
(A) Anemia
(B) Carbon monoxide poisoning
(C) Hypoventilation

(D) Low V/Q ratio


(E) Right-to-left shunt

281. A49-year-old farmer develops a headache and becomes dizzy after working on a tractor in his
barn. His wife suspects carbon monoxide poisoning and brings him to the emergency department
where he complains of dizziness, lightheadedness, headache, and nausea. The patients skin is
red, he does not appear to be in respiratory distress, and denies dyspnea. Blood levels of
carboxyhemoglobin are elevated. Which of the following best explains the absence of respiratory
signs and symptoms associated with carbon monoxide poisoning?
(A) Blood flow to the carotid body is decreased
(B) Arterial oxygen content is normal
(C) Cerebrospinal fluid pH is normal
(D) Central chemoreceptors are depressed
(E) Arterial oxygen tension is normal
282. The venous O2 tension is higher than normal in which one of the following conditions?
(A) Cyanide poisoning
(B) Exercise
(C) Decreased cardiac output

(D) Anemia
(E) CO poisoning

Respiratory Physiology
283. A scientist doing experiments with sodium cyanide started experiencing headache, dizziness,
clumsiness, decreased visual acuity, and nausea. The medical student doing research in the
laboratory was not certain if this was unusual behavior for the professor, but thought it best to
take him to the emergency department to be evaluated for possible hypoxia. Blood values
obtained on the professor while he was breathing room air were as follows:
Hb: 16 g/dL
PaO2: 102 mm Hg
PaCO2: 27 mm Hg
pH: 7.57
[HCO3]: 23 mEq/L

SaO2: 97.5%
PvO2: 65 mm Hg
SvO2: 90%
Cardiac output: 5.6 L/min

The professors hypoxia is most likely the result of which of the following?
(A) Hypoxemia
(B) Impaired hemoglobin oxygen transport
(C) Impaired oxygen delivery
(D) Impaired oxygen utilization
(E) Impaired diffusion across the alveolar-capillary membrane
284. An 83-year-old woman is found unresponsive by her son approximately 3 hours after her gall
bladder surgery. The nurse reported that the patient had asked for her pain meds and said she was
going to rest for a while. Arterial blood gases reveal hypercapnia and hypoxemia. Which of the
following is the most likely cause of the high arterial PCO2?
(A) Decreased metabolic activity
(B) Decreased alveolar dead space
(C) Hypoventilation

(D) Hypoxemia
(E) V/Q inequality

285. Oxygen therapy is most beneficial in which of the following situations? Assume lung function is
normal.
(A) Anemia
(B) CO2 retention (COPD)

(C) Cyanide poisoning


(D) High altitude

Anda mungkin juga menyukai